Cardiac NCLEX questions

Lakukan tugas rumah & ujian kamu dengan baik sekarang menggunakan Quizwiz!

Before a pt receives metoprolol (Lopressor) for hypertension, the nurse should ask the pt about a history of which of the following? A. bronchospasms B. seizures C. peripheral vascular disease D. myasthenia gravis

A

When nitrates are administered early to the acute MI pt, the result is: A. Hypotension B. Bradycardia C. reduced mortality D. reduced morbidity

C

A pt has a new prescription from transdermal nitroglycerin patches. The nurse teaches the pt that these patches are most appropriately used for which of the following? A. to relieve exertional angina B. to prevent palpatations C. to prevent the occurrence of angina D. to reduce the severity of anginal episodes

C

Which of the following characteristics is typical of the pain associated with DVT? A. Dull ache B. No pain C. Sudden onset D. Tingling

C

Which of the following is a contraindication for digoxin administration? A. BP of 140/90 B. HR>80 C. HR<60 D. RR>18

C

A pt is diagnosed with an acute myocardial infarction and is receiving tissue plasminogen activator, alteplase (Activase). Which of the following is a priority nursing intervention? A. monitor for renal failure B. monitor for psychosocial status C. monitor for signs of bleeding D. have heparin sodium available

C

which of the following is the serum abnormality with thiazides? A. hyperkalemia B. hyperglycemia C. hyperchloremia

B

The nurse is instructing a client about his high cholesterol level and wants to include behavioral considerations. Which of the following should be included in this instruction? a. The need for annual flu inoculation b. The need for an annual cholesterol panel c. The need to stop smoking d. The need to reduce stress

C

When assessing a pt who has been taking amiodarone for 6 months, which adverse reaction might the nurse identify? A. glycosuria B. dysphagia C. photophobia D. urticaria

C

You know that the action of nitrates is: A. smooth muscle contraction B. vasoconstriction C. smooth muscle relaxation D. increase preload

C

Your pt is on coumadin. You must instruct them to limit their intake of what? A. milk B. juice C. tea

C

A pulsating abdominal mass usually indicates which of the following conditions? A. Abdominal aortic aneurysm B. Enlarged spleen C. Gastic distention D. Gastritis

A The presence of a pulsating mass in the abdomen is an abnormal finding, usually indicating an outpouching in a weakened vessel, as in abdominal aortic aneurysm. The finding, however, can be normal on a thin person. Neither an enlarged spleen, gastritis, nor gastic distention cause pulsation.

a pt with congestive heart failure is receiving digoxin. What is the desired effect: A. neck vein distention B. decreased appetite C. increased urinary output D. increased pedal edema

C

during assessment of a pt who is receiving digoxin, which finding would indicate an increased possibility of toxicity? A. apical pulse rate of 60 bpm B. digoxin level of 1.5 C. serum potassium level of 2.0 D. serum potassium level of 4.8

C

positive inotropic action does which of the following? A. decreases heart rate B. decreases cellular conduction C. increases contractility

C

A nurse is caring for a client who had a resection of an abdominal aortic aneurysm yesterday. The client has an intravenous infusion with a rate of 150 mL/hr, unchanged for the last 10 hours. The client's urine output for the last 3 hours was 90, 50, and 28 mL (28 mL most recent). The client's blood urea nitrogen level is 35 mg/dL and serum creatinine level is 1.8 mg/dL, measured this morning. Which of the following actions should the nurse take next? 1. Call the physician. 2. Check the urine specific gravity. 3. Check to see if the client had a sample for serum albumin level drawn. 4. Put the intravenous line on a pump so that the infusion rate is sure to stay stable.

1 Rationale: Following abdominal aortic aneurysm resection or repair, the nurse monitors the client for signs of renal failure. Renal failure can occur because often much blood is lost during the surgery and, depending on the aneurysm location, the renal arteries may be hypoperfused for a short period during surgery. The nurse monitors hourly intake and output and notes the results of daily blood urea nitrogen and creatinine levels. Urine output lower than 30 to 50 mL/hr is reported to the physician.

Intravenous heparin therapy is prescribed for a client. While implementing this prescription, a nurse ensures that which of the following medications is available on the nursing unit? 1. Protamine sulfate 2. Potassium chloride 3. Aminocaproic acid (Amicar) 4. Vitamin K (AquaMEPHYTON)

1 Rationale: The antidote to heparin is protamine sulfate; it should be readily available for use if excessive bleeding or hemorrhage should occur. Vitamin K is an antidote for warfarin sodium. Aminocaproic acid is the antidote for thrombolytic therapy. Potassium chloride is administered for a potassium deficit

A client has frequent bursts of ventricular tachycardia on the cardiac monitor. What should the nurse be most concerned about with this dysrhythmia? 1. It can develop into ventricular fibrillation at any time. 2. It is almost impossible to convert to a normal rhythm. 3. It is uncomfortable for the client, giving a sense of impending doom. 4. It produces a high cardiac output that quickly leads to cerebral and myocardial ischemia.

1 Rationale: Ventricular tachycardia is a life-threatening dysrhythmia that results from an irritable ectopic focus that takes over as the pacemaker for the heart. The low cardiac output that results can lead quickly to cerebral and myocardial ischemia. Clients frequently experience a feeling of impending doom. Ventricular tachycardia is treated with antidysrhythmic medications, cardioversion (client awake), or defibrillation (loss of consciousness). Ventricular tachycardia can deteriorate into ventricular fibrillation at any time.

A client with rapid rate atrial fibrillation asks a nurse why the physician is going to perform carotid sinus massage. Which of the following would be reflective of a correct explanation provided by the nurse? 1. The vagus nerve slows the heart rate. 2. The diaphragmatic nerve slows the heart rate. 3. The diaphragmatic nerve overdrives the rhythm. 4. The vagus nerve increases the heart rate, overdriving the rhythm.

1 Rationale: Carotid sinus massage is one maneuver used for vagal stimulation to decrease a rapid heart rate and possibly terminate a tachydysrhythmia. The others include inducing the gag reflex and asking the client to strain or bear down. Medication therapy often is needed as an adjunct to keep the rate down or maintain the normal rhythm. Options 2, 3, and 4 are incorrect descriptions of this procedure

A nurse assesses the sternotomy incision of a client on the third day after cardiac surgery. The incision shows some slight "puffiness" along the edges and is nonreddened, with no apparent drainage. Temperature is 99° F orally. The white blood cell count is 7500 cells/mm3. How should the nurse interpret these findings? 1. Incision is slightly edematous but shows no active signs of infection. 2. Incision shows early signs of infection, although the temperature is nearly normal. 3. Incision shows early signs of infection, supported by an elevated white blood cell count. 4. Incision shows no sign of infection, although the white blood cell count is elevated.

1 Rationale: Sternotomy incision sites are assessed for signs and symptoms of infection, such as redness, swelling, induration, and drainage. Elevated temperature and white blood cell count after 3 to 4 days postoperatively usually indicate infection.

A client has frequent bursts of ventricular tachycardia on the cardiac monitor. Why should the nurse be most concerned about with this dysrhythmia? 1. It can develop into ventricular fibrillation at any time. 2. It is almost impossible to convert to a normal rhythm. 3. It is uncomfortable for the client, giving a sense of impending doom. 4. It produces a high cardiac output that quickly leads to cerebral and myocardial ischemia.

1 Rationale: Ventricular tachycardia is a life-threatening dysrhythmia that results from an irritable ectopic focus that takes over as the pacemaker for the heart. The low cardiac output that results can lead quickly to cerebral and myocardial ischemia. Clients frequently experience a feeling of impending doom. Ventricular tachycardia is treated with antidysrhythmic medications, cardioversion (client awake), or defibrillation (loss of consciousness). Ventricular tachycardia can deteriorate into ventricular fibrillation at any time.

A nurse in a medical unit is caring for a client with heart failure. The client suddenly develops extreme dyspnea, tachycardia, and lung crackles and the nurse suspects pulmonary edema. The nurse immediately asks another nurse to contact the physician and prepares to implement which priority interventions? Select all that apply. 1. Administering oxygen 2. Inserting a Foley catheter 3. Administering furosemide (Lasix) 4. Administering morphine sulfate intravenously 5. Transporting the client to the coronary care unit 6. Placing the client in a low Fowler's side-lying position

1, 2, 3, 4 Rationale: Pulmonary edema is a life-threatening event that can result from severe heart failure. In pulmonary edema, the left ventricle fails to eject sufficient blood, and pressure increases in the lungs because of the accumulated blood. Oxygen is always prescribed, and the client is placed in a high Fowler's position to ease the work of breathing. Furosemide, a rapid-acting diuretic, will eliminate accumulated fluid. A Foley catheter is inserted to measure output accurately. Intravenously administered morphine sulfate reduces venous return (preload), decreases anxiety, and also reduces the work of breathing. Transporting the client to the coronary care unit is not a priority intervention. In fact, this may not be necessary at all if the client's response to treatment is successful.

A nurse is monitoring the digoxin level for a client who has been taking a daily dose of digoxin for 1 month. the digoxin level is 0.25 ng/mL. The nurse should notify the provider and anticipate which of the following: 1. An increase in the client's digoxin dose. 2. A decrease in the client's digoxin dose. 3. No change in the client's digoxin dose. 4. Dicontinuation of the client's digoxin dose.

1. An increase in the client's digoxin dose.

A nurse is providing teaching to a client who has a new prescription for digoxin (Lanoxin) Which of the following may indicate dig toxicity & should be reported to the provider? 1. Fatigue 2. constipation 3. Anorexia 4. Rash 5. Diplopia

1. Fatigue Not constipation but -- nausea, vomiting & diarrhea 3. Anorexia b/c GI disturbances 5. Diplopia -- visual changes , halo, yellow-tinged vision.

A client with myocardial infarction is developing cardiogenic shock. Because of the risk of myocardial ischemia, for which of the following should the nurse carefully assess the client? 1. Bradycardia 2. Ventricular dysrhythmias 3. Rising diastolic blood pressure 4. Falling central venous pressure

2 Rationale: Classic signs of cardiogenic shock as they relate to this question include low blood pressure and tachycardia. The central venous pressure would rise as the backward effects of the severe left ventricular failure became apparent. Dysrhythmias commonly occur as a result of decreased oxygenation and severe damage to greater than 40% of the myocardium.

A client with atrial fibrillation is receiving a continuous heparin infusion at 1000 units/hr. The nurse would determine that the client is receiving the therapeutic effect based on which of the following results? 1. Prothrombin time of 12.5 seconds 2. Activated partial thromboplastin time of 60 seconds 3. Activated partial thromboplastin time of 28 seconds 4. Activated partial thromboplastin time longer than 120 seconds

2 Rationale: Common laboratory ranges for activated partial thromboplastin time are 20 to 36 seconds. Because the activated partial thromboplastin time should be 1.5 to 2.5 times the normal value, the client's activated partial thromboplastin time would be considered therapeutic if it was 60 seconds.

A client with myocardial infarction suddenly becomes tachycardic, shows signs of air hunger, and begins coughing frothy, pink-tinged sputum. Which of the following would the nurse anticipate when auscultating the client's breath sounds? 1. Stridor 2. Crackles 3. Scattered rhonchi 4. Diminished breath sounds

2 Rationale: Pulmonary edema is characterized by extreme breathlessness, dyspnea, air hunger, and the production of frothy, pink-tinged sputum. Auscultation of the lungs reveals crackles. Rhonchi and diminished breath sounds are not associated with pulmonary edema. Stridor is a crowing sound associated with laryngospasm or edema of the upper airway.

A client who had cardiac surgery 24 hours ago has a urine output averaging 20 mL/hr for 2 hours. The client received a single bolus of 500 mL of intravenous fluid. Urine output for the subsequent hour was 25 mL. Daily laboratory results indicate that the blood urea nitrogen level is 45 mg/dL and the serum creatinine level is 2.2 mg/dL. Based on these findings, the nurse would anticipate that the client is at risk for which of the following? 1. Hypovolemia 2. Acute renal failure 3. Glomerulonephritis 4. Urinary tract infection

2 Rationale: The client who undergoes cardiac surgery is at risk for renal injury from poor perfusion, hemolysis, low cardiac output, or vasopressor medication therapy. Renal insult is signaled by decreased urine output and increased blood urea nitrogen and creatinine levels. The client may need medications to increase renal perfusion and possibly could need peritoneal dialysis or hemodialysis. No data in the question indicate the presence of hypovolemia, urinary tract infection, or glomerulonephritis.

A client with angina complains that the anginal pain is prolonged and severe and occurs at the same time each day, most often at rest in the absence of precipitating factors. How would the nurse best describe this type of anginal pain? 1. Stable angina 2. Variant angina 3. Unstable angina 4. Nonanginal pain

2 Rationale: Variant angina, or Prinzmetal's angina, is prolonged and severe and occurs at the same time each day, most often at rest. Stable angina is induced by exercise and relieved by rest or nitroglycerin tablets. Unstable angina occurs at lower and lower levels of activity or at rest, is less predictable, and is often a precursor of myocardial infarction.

nurse is evaluating a client's response to cardioversion. Which of the following observations would be of highest priority to the nurse? 1. Blood pressure 2. Status of airway 3. Oxygen flow rate 4. Level of consciousness

2 Rationale: Nursing responsibilities after cardioversion include maintenance first of a patent airway, and then oxygen administration, assessment of vital signs and level of consciousness, and dysrhythmia detection.

The nurse is monitoring a client who is taking digoxin (Lanoxin) for adverse effects. Which findings are characteristic of digoxin toxicity. Select all that apply. 1. Tremors 2. Diarrhea 3. Irritability 4. Blurred vision 5. Nausea and vomiting

2, 4, 5 Rationale: Digoxin (Lanoxin) is a cardiac glycoside. The risk of toxicity can occur with the use of this medication. Toxicity can lead to life-threatening events and the nurse needs to monitor the client closely for signs of toxicity. Early signs of toxicity include gastrointestinal manifestations such as anorexia, nausea, vomiting, and diarrhea. Subsequent manifestations include headache, visual disturbances such as diplopia, blurred vision, yellow-green halos, photophobia, drowsiness, fatigue, and weakness. Cardiac rhythm abnormalities can also occur. The nurse also monitors the digoxin level. Therapeutic levels for digoxin range from 0.5 to 2 ng/mL.

A nurse is administering a dopamine infusion at a moderate dose to a client who has severe HF. Which of the following is an expected effect? 1. Lowered heart rate 2. Increased myocardial contractility 3. Decreased conduction through the AV node D. Vasoconstriction of the renal blood vessels

2. Increased myocardial contractility -- thus increasing CO

A 66-year-old client complaining of not feeling well is seen in a clinic. The client is taking several medications for the control of heart disease and hypertension. These medications include atenolol (Tenormin), digoxin (Lanoxin), and chlorothiazide (Diuril). A tentative diagnosis of digoxin toxicity is made. Which of the following assessment data would support this diagnosis? 1. Dyspnea, edema, and palpitations 2. Chest pain, hypotension, and paresthesia 3. Double vision, loss of appetite, and nausea 4. Constipation, dry mouth, and sleep disorder

3 Rationale: Double vision, loss of appetite, and nausea are early signs of digoxin toxicity. Additional signs of digoxin toxicity include bradycardia, difficulty reading, other visual alterations such as green and yellow vision or seeing spots or halos, confusion, vomiting, diarrhea, decreased libido, and impotence.

A client is wearing a continuous cardiac monitor, which begins to sound its alarm. A nurse sees no electrocardiographic complexes on the screen. Which of the following should be the priority action of the nurse? 1. Call a code blue. 2. Call the physician. 3. Check the client status and lead placement. 4. Press the recorder button on the electrocardiogram console.

3 Rationale: Sudden loss of electrocardiographic complexes indicates ventricular asystole or possibly electrode displacement. Accurate assessment of the client and equipment is necessary to determine the cause and identify the appropriate intervention. Options 1, 2, and 4 are unnecessary

A nurse is planning to administer hydrochlorothiazide (HydroDIURIL) to a client. The nurse understands that which of the following are concerns related to the administration of this medication? 1. Hypouricemia, hyperkalemia 2. Increased risk of osteoporosis 3. Hypokalemia, hyperglycemia, sulfa allergy 4. Hyperkalemia, hypoglycemia, penicillin allergy

3 Rationale: Thiazide diuretics such as hydrochlorothiazide are sulfa-based medications, and a client with a sulfa allergy is at risk for an allergic reaction. Also, clients are at risk for hypokalemia, hyperglycemia, hypercalcemia, hyperlipidemia, and hyperuricemia.

A client is admitted with pulmonary embolism and is to be treated with streptokinase (Streptase). A nurse would report which of the following assessments to the physician before initiating this therapy? 1. Adventitious breath sounds 2. Temperature of 99.4° F orally 3. Blood pressure of 198/110 mm Hg 4. Respiratory rate of 28 breaths/min

3 Rationale: Thrombolytic therapy is contraindicated in a number of preexisting conditions in which there is a risk of uncontrolled bleeding, similar to the case in anticoagulant therapy. Thrombolytic therapy also is contraindicated in severe uncontrolled hypertension because of the risk of cerebral hemorrhage. Therefore the nurse would report the results of the blood pressure to the physician before initiating therapy.

A client is diagnosed with an acute myocardial infarction and is receiving tissue plasminogen activator, alteplase (Activase, tPA). Which of the following is a priority nursing intervention? 1. Monitor for renal failure. 2. Monitor psychosocial status. 3. Monitor for signs of bleeding. 4. Have heparin sodium available

3 Rationale: Tissue plasminogen activator is a thrombolytic. Hemorrhage is a complication of any type of thrombolytic medication. The client is monitored for bleeding. Monitoring for renal failure and monitoring the client's psychosocial status are important but are not the most critical interventions. Heparin is given after thrombolytic therapy, but the question is not asking about follow-up medications.

A home health care nurse is visiting an older client at home. Furosemide (Lasix) is prescribed for the client and the nurse teaches the client about the medication. Which of the following statements, if made by the client, indicates the need for further teaching? 1. "I will sit up slowly before standing each morning." 2. "I will take my medication every morning with breakfast." 3. "I need to drink lots of coffee and tea to keep myself healthy." 4. "I will call my doctor if my ankles swell or my rings get tight."

3 Rationale: Tea and coffee are stimulants and mild diuretics. These are a poor choice for hydration. Taking the medication at the same time each day improves compliance. Because furosemide is a diuretic, the morning is the best time to take the medication so as not to interrupt sleep. Notification of the health care provider is appropriate if edema is noticed in the hands, feet, or face or if the client is short of breath. Sitting up slowly prevents postural hypotension.

A nurse is preparing to ambulate a client on the third day after cardiac surgery. The nurse would plan to do which of the following to enable the client to best tolerate the ambulation? 1. Remove telemetry equipment. 2. Provide the client with a walker. 3. Premedicate the client with an analgesic. 4. Encourage the client to cough and deep breathe.

3 Rationale: The nurse should encourage regular use of pain medication for the first 48 to 72 hours after cardiac surgery because analgesia will promote rest, decrease myocardial oxygen consumption resulting from pain, and allow better participation in activities such as coughing, deep breathing, and ambulation. Options 2 and 4 will not help in tolerating ambulation. Removal of telemetry equipment is contraindicated unless prescribed.

When teaching a patient with peripheral arterial disease, the nurse determines that further teaching is needed when the patient says, 1. "I should not use heating pads to warm my feet." 2. "I will examine my feet every day for any sores or red areas." 3. "I should cut back on my walks if they cause pain in my legs." 4. "I think I can quit smoking with the use of short-term nicotine replacement and support groups."

3. "I should cut back on my walks if they cause pain in my legs." (Rationale: Patients should be taught to exercise to the point of discomfort, stop and rest, and then resume walking until the discomfort recurs. Smoking cessation and proper foot care are also important interventions for patients with peripheral arterial disease.)

A client with HF has an order for lisnopril (Prinivil, Zestril) Which of the following conditions in the client's history would lead a nurse to confirm the order with the provider? 1. A history of HT previously treated with diuretics. 2. A history of seasonal allergies currently treated with antihistamines. 3. A history of angioedema after taking enalapril (Vasotec) 4. A history of alcoholism, currently abstaining.

3. A history of angioedema after taking enalapril (Vasotec)

A client admitted to the hospital with chest pain and history of type 2 diabetes mellitus is scheduled for cardiac catheterization. Which of the following medications would need to be withheld for 48 hours before and after the procedure? 1. Regular insulin 2. Glipizide (Glucotrol) 3. Repaglinide (Prandin) 4. Metformin (Glucophage)

4 Rationale: Metformin (Glucophage) needs to be withheld 48 hours before and after cardiac catheterization because of the injection of contrast medium during the procedure. If the contrast medium affects kidney function, with metformin in the system, the client would be at increased risk for lactic acidosis. The medications in options 1, 2, and 3 do not need to be withheld 48 hours before or after cardiac catheterization.

the pt states, "I always put my nitroglycerine patch in the same place so I do not forget to take it off." Which of the following is the best response by the nurse? A. change the patch every 24 hours B. massage into the skin if ointment comes in contact with your hands C. rotate the NTG patch to a different hairless area each day D. after removing the patch, scrub the are vigorously with soap and water

C

A nurse provides discharge instructions to a postoperative client who is taking warfarin sodium (Coumadin). Which statement, if made by the client, reflects the need for further teaching? 1. "I will take my pills every day at the same time." 2. "I will avoid alcohol consumption." 3. "I have already called my family to pick up a Medic-Alert bracelet." 4. "I will take Ecotrin (enteric-coated aspirin) for my headaches because it is coated."

4 Rationale: Ecotrin is an aspirin-containing product and should be avoided. Alcohol consumption should be avoided by a client taking warfarin sodium. Taking prescribed medication at the same time each day increases client compliance. The Medic-Alert bracelet provides health care personnel emergency information.

A client is on nicotinic acid (niacin) for hyperlipidemia and the nurse provides instructions to the client about the medication. Which statement by the client would indicate an understanding of the instructions? 1. "It is not necessary to avoid the use of alcohol." 2. "The medication should be taken with meals to decrease flushing. 3. "Clay-colored stools are a common side effect and should not be of concern." 4. "Ibuprofen (Motrin) taken 30 minutes before the nicotinic acid should decrease the flushing."

4 Rationale: Flushing is a side effect of this medication. Aspirin or a nonsteroidal anti-inflammatory drug can be taken 30 minutes prior to taking the medication to decrease flushing. Alcohol consumption needs to be avoided because it will enhance this side effect. The medication should be taken with meals but this will decrease gastrointestinal upset; taking the medication with meals has no effect on the flushing. Clay-colored stools are a sign of hepatic dysfunction and should be immediately reported to the physician.

A client is in sinus bradycardia with a heart rate of 45 beats/min, complains of dizziness, and has a blood pressure of 82/60 mm Hg. Which of the following should the nurse anticipate will be prescribed? 1. Defibrillate the client. 2. Administer digoxin (Lanoxin). 3. Continue to monitor the client. 4. Prepare for transcutaneous pacing

4 Rationale: Hypotension and dizziness are signs of decreased cardiac output. Transcutaneous pacing provides a temporary measure to increase the heart rate and thus perfusion in the symptomatic client. Digoxin will further decrease the client's heart rate. Defibrillation is used for treatment of pulseless ventricular tachycardia and ventricular fibrillation. Continuing to monitor the client delays necessary intervention

A home health care nurse is visiting a client with elevated triglyceride levels and a serum cholesterol level of 398 mg/dL. The client is taking cholestyramine (Questran). Which of the following statements, if made by the client, indicates the need for further education? 1. "Constipation and bloating might be a problem." 2. "I'll continue to watch my diet and reduce my fats." 3. "Walking a mile each day will help the whole process." 4. "I'll continue my nicotinic acid from the health food store."

4 Rationale: Nicotinic acid, even an over-the-counter form, should be avoided because it may lead to liver abnormalities. All lipid-lowering medications also can cause liver abnormalities, so a combination of nicotinic acid and cholestyramine resin is to be avoided. Constipation and bloating are the two most common side effects. Walking and the reduction of fats in the diet are therapeutic measures to reduce cholesterol and triglyceride levels.

A client is having frequent premature ventricular contractions. A nurse would place priority on assessment of which of the following? 1. Sensation of palpitations 2. Causative factors, such as caffeine 3. Precipitating factors, such as infection 4. Blood pressure and oxygen saturation

4 Rationale: Premature ventricular contractions can cause hemodynamic compromise. The shortened ventricular filling time with the ectopic beat leads to decreased stroke volume and, if frequent enough, to decreased cardiac output. The client may be asymptomatic or may feel palpitations. Premature ventricular contractions can be caused by cardiac disorders, states of hypoxemia, or by any number of physiological stressors, such as infection, illness, surgery, or trauma, and by intake of caffeine, nicotine, or alcohol.

A client is having frequent premature ventricular contractions. A nurse would place priority on assessment of which of the following? 1. Sensation of palpitations 2. Causative factors, such as caffeine 3. Precipitating factors, such as infection 4. Blood pressure and oxygen saturation

4 Rationale: Premature ventricular contractions can cause hemodynamic compromise. The shortened ventricular filling time with the ectopic beat leads to decreased stroke volume and, if frequent enough, to decreased cardiac output. The client may be asymptomatic or may feel palpitations. Premature ventricular contractions can be caused by cardiac disorders, states of hypoxemia or by any number of physiological stressors, such as infection, illness, surgery, or trauma, and by intake of caffeine, nicotine, or alcohol

when a pt is being taught about the potential adverse effects of an ACE inhibitor, which of the following should be mentioned as possibly occurring when this drug is taken to treat hypertension A. hypokalemia B. nausea C. dry, nonproductive cough D. sedation

C

A 50 y/o pt is prescribed to take nitrate each day for his condition. You know the result of nitrate administration is: A. decreased myocardial oxygen demand B. increased myocardial oxygen demand C. increased left ventricular end-diastolic volume D. increased atrial pressure

A

A nurse is assessing the neurovascular status of a client who returned to the surgical nursing unit 4 hours ago after undergoing aortoiliac bypass graft. The affected leg is warm, and the nurse notes redness and edema. The pedal pulse is palpable and unchanged from admission. The nurse interprets that the neurovascular status is: A. Normal because of the increased blood flow through the leg B. Slightly deteriorating and should be monitored for another hour C. Moderately impaired, and the surgeon should be called. D. Adequate from the arterial approach, but venous complications are arising.

A

A patient who is in the 10th week of outpatient cardiac rehabilitation continues to exhibit symptoms of depression. When developing a discharge plan, the cardiac/vascular nurse includes: A. a referral for counseling and possible medication. B. a trial of herbal remedies. C. alternative therapies, including yoga and massage. D. comprehensive information about the patient's cardiac status to help reduce his or her anxiety.

A

A pt has a potassium level of 6.0 and a digoxin level of 3.0. What medication would the nurse be giving: A. digibind B. sodium citrate C. epinephrine D. lidocaine

A

Mike, a 43-year old construction worker, has a history of hypertension. He smokes two packs of cigarettes a day, is nervous about the possibility of being unemployed, and has difficulty coping with stress. His current concern is calf pain during minimal exercise that decreased with rest. The nurse assesses Mike's symptoms as being associated with peripheral arterial occlusive disease. The nursing diagnosis is probably: A. Alteration in tissue perfusion related to compromised circulation B. Dysfunctional use of extremities related to muscle spasms C. Impaired mobility related to stress associated with pain D. Impairment in muscle use associated with pain on exertion

A

Raymund is reviewing on cardio drugs for his upcoming exam. For a well-prepared student, he should know that vasodilators are agents that: A. relax smooth muscle B. are used to treat hypotension C. stimulate the adrenergic receptors D. cause respiratory depression

A

When applying a nitro patch it should be placed on A. non hairy part of the chest B. thigh C. upper back D. buttocks

A

When teaching about nitrate administration, the nurse should instruct the pt to: A. change positions slowly B. take pulse weekly C. reduce salt intake D. chew the sustained-release tablets

A

Which is the most important action for the nurse to take before administering digoxin? A. monitor the potassium level B. assess the BP C. evaluate urinary output D. avoid giving with a thiazide diuretic

A

Which statement by the pt reflects the need for additional pt education about the CCB diltiazem (Cardizem)? A. I can take this drug to stop acute anginal attacks B. I understand that food and antacids alter the absorption of this oral drug C. when the long-acting forms are taken, the drug cannot be crushed D. this drug may cause my blood pressure to drop, so I should be careful when getting up

A

Your pt starts showing signs of gynecomastia, which diuretic would you suspect they are on? A. potassium-sparing B. thiazide C. loop D. calcium channel blockers

A

beta blockers should be avoided in which of the following conditions: A. bronchoconstriction B. hypertension C. Angina D. myocardial infarction

A

Which of the following instructions should be included in the discharge teaching for a patient discharged with a transdermal nitroglycerin patch? A. "Apply the patch to a non hairy, nonfatty area of the upper torso or arms." B. "Apply the patch to the same site each day to maintain consistent drug absorption." C. "If you get a headache, remove the patch for 4 hours and then reapply." D. "If you get chest pain, apply a second patch right next to the first patch.

A A nitroglycerin patch should be applied to a non hairy, nonfatty area for the best and most consistent absorption rates. Sites should be rotated to prevent skin irritation, and the drug should be continued if headache occurs because tolerance will develop. Sublingual nitroglycerin should be used to treat chest pain.

A 60-year-old male client comes into the emergency department with complaints of crushing chest pain that radiates to his shoulder and left arm. The admitting diagnosis is acute myocardial infarction. Immediate admission orders include oxygen by NC at 4L/minute, blood work, chest x-ray, an ECG, and 2 mg of morphine given intravenously. The nurse should first: A. Administer the morphine B. Obtain a 12-lead ECG C. Obtain the lab work D. Order the chest x-ray

A Although obtaining the ECG, chest x-ray, and blood work are all important, the nurse's priority action would be to relieve the crushing chest pain.

A nurse notes 2+ bilateral edema in the lower extremities of a client with myocardial infarction who was admitted 2 days ago. The nurse would plan to do which of the following next? A. Review the intake and output records for the last 2 days B. Change the time of diuretic administration from morning to evening C. Request a sodium restriction of 1 g/day from the physician D. Order daily weights starting the following morning

A Edema, the accumulation of excess fluid in the interstitial spaces, can be measured by intake greater than output and by a sudden increase in weight. Diuretics should be given in the morning whenever possible to avoid nocturia. Strict sodium restrictions are reserved for clients with severe symptoms.

Following a treadmill test and cardiac catheterization, the client is found to have coronary artery disease, which is inoperative. He is referred to the cardiac rehabilitation unit. During his first visit to the unit he says that he doesn't understand why he needs to be there because there is nothing that can be done to make him better. The best nursing response is: A. "Cardiac rehabilitation is not a cure but can help restore you to many of your former activities." B. "Here we teach you to gradually change your lifestyle to accommodate your heart disease." C. "You are probably right but we can gradually increase your activities so that you can live a more active life." D. "Do you feel that you will have to make some changes in your life now?

A Such a response does not have false hope to the client but is positive and realistic. The answer tells the client what cardiac rehabilitation is and does not dwell upon his negativity about it.

Which of the following symptoms should the nurse teach the client with unstable angina to report immediately to her physician? A. A change in the pattern of her pain B. Pain during sex C. Pain during an argument with her husband D. Pain during or after an activity such as lawn mowing

A The client should report a change in the pattern of chest pain. It may indicate increasing severity of CAD.

Which of the following sounds is distinctly heard on auscultation over the abdominal region of an abdominal aortic aneurysm client? A. Bruit B. Crackles C. Dullness D. Friction rubs

A A bruit, a vascular sound resembling heart murmur, suggests partial arterial occlusion. Crackles are indicative of fluid in the lungs. Dullness is heard over solid organs, such as the liver. Friction rubs indicate inflammation of the peritoneal surface.

What is the most common symptom in a client with abdominal aortic aneurysm? A. Abdominal pain B. Diaphoresis C. Headache D. Upper back pain

A Abdominal pain in a client with an abdominal aortic aneurysm results from the disruption of normal circulation in the abdominal region. Lower back pain, not upper, is a common symptom, usually signifying expansion and impending rupture of the aneurysm. Headache and diaphoresis aren't associated with abdominal aortic aneurysm.

Which of the following types of cardiomyopathy can be associated with childbirth? A. Dilated B. Hypertrophic C. Myocarditis D. Restrictive

A Although the cause isn't entirely known, cardiac dilation and heart failure may develop during the last month of pregnancy of the first few months after birth. The condition may result from a preexisting cardiomyopathy not apparent prior to pregnancy. Hypertrophic cardiomyopathy is an abnormal symmetry of the ventricles that has an unknown etiology but a strong familial tendency. Myocarditis isn't specifically associated with childbirth. Restrictive cardiomyopathy indicates constrictive pericarditis; the underlying cause is usually myocardial.

Which of the following parameters should be checked before administering digoxin? A. Apical pulse B. Blood pressure C. Radial pulse D. Respiratory rate

A An apical pulse is essential or accurately assessing the client's heart rate before administering digoxin. The apical pulse is the most accurate point in the body. Blood pressure is usually only affected if the heart rate is too low, in which case the nurse would withhold digoxin. The radial pulse can be affected by cardiac and vascular disease and therefore, won't always accurately depict the heart rate. Digoxin has no effect on respiratory function.

What is the most common cause of abdominal aortic aneurysm? A. Atherosclerosis B. DM C. .HPN D. Syphilis

A Atherosclerosis accounts for 75% of all abdominal aortic aneurysms. Plaques build up on the wall of the vessel and weaken it, causing an aneurysm. Although the other conditions are related to the development of an aneurysm, none is a direct cause.

Which of the following conditions most commonly results in CAD? A. Atherosclerosis B.DM C. MI D. Renal failure

A Atherosclerosis, or plaque formation, is the leading cause of CAD. DM is a risk factor for CAD but isn't the most common cause. Renal failure doesn't cause CAD, but the two conditions are related. Myocardial infarction is commonly a result of CAD.

Which of the following recurring conditions most commonly occurs in clients with cardiomyopathy? A. Heart failure B. DM C. MI D. Pericardial effusion

A Because the structure and function of the heart muscle is affected, heart failure most commonly occurs in clients with cardiomyopathy. Myocardial infarction results from prolonged myocardial ischemia due to reduced blood flow through one of the coronary arteries. Pericardial effusion is most predominant in clients with percarditis. Diabetes mellitus is unrelated to cardiomyopathy.

Which of the following classes of medications protects the ischemic myocardium by blocking catecholamines and sympathetic nerve stimulation? A. Beta-adrenergic blockers B. Calcium channel blockers C. Narcotics D. Nitrates

A Beta-adrenergic blockers work by blocking beta receptors in the myocardium, reducing the response to catecholamines and sympathetic nerve stimulation. They protect the myocardium, helping to reduce the risk of another infarction by decreasing the workload of the heart and decreasing myocardial oxygen demand. Calcium channel blockers reduce the workload of the heart by decreasing the heart rate. Narcotics reduce myocardial oxygen demand, promote vasodilation, and decreased anxiety. Nitrates reduce myocardial oxygen consumption by decreasing left ventricular end-diastolic pressure (preload) and systemic vascular resistance (afterload).

What is the term used to describe an enlargement of the heart muscle? A. Cardiomegaly B. Cardiomyopathy C. Myocarditis D. Pericarditis

A Cardiomegaly denotes an enlarged heart muscle. Cardiomyopathy is a heart muscle disease of unknown origin. Myocarditis refers to inflammation of heart muscle. Pericarditis is an inflammation of the pericardium, the sac surrounding the heart.

Which of the following heart muscle diseases is unrelated to other cardiovascular disease? A. Cardiomyopathy B. Coronary artery disease C. Myocardial infarction D. Pericardial Effusion

A Cardiomyopathy isn't usually related to an underlying heart disease such as atherosclerosis. The etiology in most cases is unknown. Coronary artery disease and myocardial infarction are directly related to atherosclerosis. Pericardial effusion is the escape of fluid into the pericardial sac, a condition associated with pericarditis and advanced heart failure.

A client with rapid rate atrial fibrillation asks a nurse why the physician is going to perform carotid massage. The nurse responds that this procedure may stimulate the: A. Vagus nerve to slow the heart rate B. Vagus nerve to increase the heart rate; overdriving the rhythm C. Diaphragmatic nerve to slow the heart rate D. Diaphragmatic nerve to overdrive the rhythm

A Carotid sinus massage is one of the maneuvers used for vagal stimulation to decrease a rapid heart rate and possibly terminate a tachydysrhythmia. The others include inducing the gag reflex and asking the client to strain or bear down. Medication therapy often is needed as an adjunct to keep the rate down or maintain the normal rhythm.

Which of the following symptoms is most commonly associated with left-sided heart failure? A. Crackles B. Arrhythmias C. Hepatic engorgement D. Hypotension

A Crackles in the lungs are a classic sign of left-sided heart failure. These sounds are caused by fluid backing up into the pulmonary system. Arrhythmias can be associated with both right and left-sided heart failure. Left-sided heart failure causes hypertension secondary to an increased workload on the system.

Good dental care is an important measure in reducing the risk of endocarditis. A teaching plan to promote good dental care in a client with mitral stenosis should include demonstration of the proper use of: A. A manual toothbrush B. An electric toothbrush C. An irrigation device D. Dental floss

A Daily dental care and frequent checkups by a dentist who is informed about the client's condition are required to maintain good oral health. Use of an electric toothbrush, an irrigation device, or dental floss may cause gums to bleed and allow bacteria to enter mucous membranes and the bloodstream, increasing the risk of endocarditis.

Toxicity from which of the following medications may cause a client to see a green halo around lights? A. Digoxin B. Furosemide C. Metoprolol D. Enalapril

A One of the most common signs of digoxin toxicity is the visual disturbance known as the green halo sign. The other medications aren't associated with such an effect.

Which of the following is the most common symptom of myocardial infarction? A. Chest pain B. Dyspnea C. Edema D. Palpitations

A The most common symptom of an MI is chest pain, resulting from deprivation of oxygen to the heart. Dyspnea is the second most common symptom, related to an increase in the metabolic needs of the body during an MI. Edema is a later sign of heart failure, often seen after an MI. Palpitations may result from reduced cardiac output, producing arrhythmias.

A client is having frequent premature ventricular contractions. A nurse would place priority on assessment of which of the following items? A. Blood pressure and peripheral perfusion B. Sensation of palpitations C. Causative factors such as caffeine D. Precipitating factors such as infection

A Premature ventricular contractions can cause hemodynamic compromise. The shortened ventricular filling time with the ectopic beats leads to decreased stroke volume and, if frequent enough, to decreased cardiac output. The client may be asymptomatic or may feel palpitations. PVCs can be caused by cardiac disorders or by any number of physiological stressors, such as infection, illness, surgery, or trauma, and by the intake of caffeine, alcohol, or nicotine.

Because a client has mitral stenosis and is a prospective valve recipient, the nurse preoperatively assesses the client's past compliance with medical regimens. Lack of compliance with which of the following regimens would pose the greatest health hazard to this client? A. Medication therapy B. Diet modification C. Activity Restrictions D. Dental care

A Preoperatively, anticoagulants may be prescribed for the client with advanced valvular heart disease to prevent emboli. Post-op, all clients with mechanical valves and some with bioprostheses are maintained indefinitely on anticoagulation therapy. Adhering strictly to a dosage schedule and observing specific precautions are necessary to prevent hemorrhage or thromboembolism. Some clients are maintained on lifelong antibiotic prophylaxis to prevent recurrence from rheumatic fever. Episodic prophylaxis is required to prevent infective endocarditis after dental procedures or upper respiratory, GI, or GU surgery.

Which of the following complications is indicated by a third heart sound (S3)? A. Ventricular dilation B. Systemic hypertension C. Aortic valve malfunction D. Increased atrial contractions

A Rapid filling of the ventricles causes vasodilation that is auscultated as S3. Increased atrial contraction or systemic hypertension can result is a fourth heart sound. Aortic valve malfunction is heard as a murmur.

A patient in the intensive care unit with ADHF complains of severe dyspnea and is anxious, tachypneic, and tachycardic. All these medications have been ordered for the patient. The first action by the nurse will be to a. administer IV morphine sulfate 2 mg. b. give IV diazepam (Valium) 2.5 mg. c. increase dopamine (Intropin) infusion by 2 mcg/kg/min. d. increase nitroglycerin (Tridil) infusion by 5 mcg/min.

A Rationale: Morphine improves alveolar gas exchange, improves cardiac output by reducing ventricular preload and afterload, decreases anxiety, and assists in reducing the subjective feeling of dyspnea. Diazepam may decrease patient anxiety, but it will not improve the cardiac output or gas exchange. Increasing the dopamine may improve cardiac output but will also increase the heart rate and myocardial oxygen consumption. Nitroglycerin will improve cardiac output and may be appropriate for this patient, but it will not directly reduce anxiety and will not act as quickly as morphine to decrease dyspnea

A patient with ADHF who is receiving nesiritide (Natrecor) asks the nurse how the medication will work to help improve the symptoms of dyspnea and orthopnea. The nurse's reply will be based on the information that nesiritide will a. dilate arterial and venous blood vessels, decreasing ventricular preload and afterload. b. improve the ability of the ventricular myocardium to contract, strengthening contractility. c. enhance the speed of impulse conduction through the heart, increasing the heart rate. d. increase calcium sensitivity in vascular smooth muscle, boosting systemic vascular resistance.

A Rationale: Nesiritide, a recombinant form of BNP, causes both arterial and venous vasodilation, leading to reductions in preload and afterload. Inotropic medications, such as dopamine and dobutamine, may be used in ADHF to improve ventricular contractility. Nesiritide does not increase impulse conduction or calcium sensitivity in the heart.

An outpatient who has developed heart failure after having an acute myocardial infarction has a new prescription for carvedilol (Coreg). After 2 weeks, the patient returns to the clinic. The assessment finding that will be of most concern to the nurse is that the patient a. has BP of 88/42. b. has an apical pulse rate of 56. c. complains of feeling tired. d. has 2+ pedal edema.

A Rationale: The patient's BP indicates that the dose of carvedilol may need to be decreased because the mean arterial pressure is only 57. Bradycardia is a frequent adverse effect of -Adrenergic blockade, but the rate of 56 is not as great a concern as the hypotension. -adrenergic blockade will initially worsen symptoms of heart failure in many patients, and patients should be taught that some increase in symptoms, such as fatigue and edema, is expected during the initiation of therapy with this class of drugs.

A patient with a history of chronic heart failure is admitted to the emergency department with severe dyspnea and a dry, hacking cough. The patient has pitting edema in both ankles, blood pressure (BP) of 170/100, an apical pulse rate of 92, and respirations 28. The most important assessment for the nurse to accomplish next is to a. auscultate the lung sounds. b. assess the orientation. c. check the capillary refill. d. palpate the abdomen.

A Rationale: When caring for a patient with severe dyspnea, the nurse should use the ABCs to guide initial care. This patient's severe dyspnea and cough indicate that acute decompensated heart failure (ADHF) is occurring. ADHF usually manifests as pulmonary edema, which should be detected and treated immediately to prevent ongoing hypoxemia and cardiac/respiratory arrest. The other assessments will provide useful data about the patient's volume status and should also be accomplished rapidly, but detection (and treatment) of fluid-filled alveoli is the priority.

A client has developed atrial fibrillation, which a ventricular rate of 150 beats per minute. A nurse assesses the client for: A. Hypotension and dizziness B. Nausea and vomiting C. Hypertension and headache D. Flat neck veins

A The client with uncontrolled atrial fibrillation with a ventricular rate more than 150 beats a minute is at risk for low cardiac output because of loss of atrial kick. The nurse assesses the client for palpitations, chest pain or discomfort, hypotension, pulse deficit, fatigue, weakness, dizziness, syncope, shortness of breath, and distended neck veins.

After an anterior wall myocardial infarction, which of the following problems is indicated by auscultation of crackles in the lungs? A. Left-sided heart failure B. Pulmonic valve malfunction C. Right-sided heart failure D. Tricuspid valve malfunction

A The left ventricle is responsible for the most of the cardiac output. An anterior wall MI may result in a decrease in left ventricular function. When the left ventricle doesn't function properly, resulting in left-sided heart failure, fluid accumulates in the interstitial and alveolar spaces in the lungs and causes crackles. Pulmonic and tricuspid valve malfunction causes right-sided heart failure.

What is the most appropriate nursing response to a myocardial infarction client who is fearful of dying? A. "Tell me about your feeling right now." B. "When the doctor arrives, everything will be fine." C. "This is a bad situation, but you'll feel better soon." D. "Please be assured we're doing everything we can to make you feel better."

A Validation of the client's feelings is the most appropriate response. It gives the client a feeling of comfort and safety. The other three responses give the client false hope. No one can determine if a client experiencing MI will feel or get better and therefore, these responses are inappropriate.

Which of the following signs and symptoms would most likely be found in a client with mitral regurgitation? A. Exertional dyspnea B. Confusion C. Elevated creatine phosphokinase concentration D. Chest pain

A Weight gain, due to fluid retention and worsening heart failure, causes exertional dyspnea in clients with mitral regurgitation. The rise in left atrial pressure that accompanies mitral valve disease is transmitted backward into pulmonary veins, capillaries, and arterioles and eventually to the right ventricle. Signs and symptoms of pulmonary and systemic venous congestion follow.

A nurse is assessing an electrocardiogram rhythm strip. The P waves and QRS complexes are regular. The PR interval is 0.16 second, and QRS complexes measure 0.06 second. The overall heart rate is 64 beats per minute. The nurse assesses the cardiac rhythm as: A. Normal sinus rhythm B. Sinus bradycardia C. Sick sinus syndrome D. First-degree heart block

A measurements are normal, measuring 0.12 to 0.20 second and 0.4 to 0.10 second, respectively.

The nurse plans close monitoring for the patient during electrophysiologic testing because this test A. requires the use of dyes that irritate the myocardium. B. causes myocardial ischemia resulting in dysrhythmias. C. induces dysrhythmias that may require defibrillation to correct. D. involves the use of anticoagulants to prevent thrombus and embolism.

C. induces dysrhythmias that may require defibrillation to correct.

What criteria should the nurse use to determine normal sinus rhythm for a client on a cardiac monitor? Check all that apply. A. The RR intervals are relatively consistent B. One P wave precedes each QRS complex C. Four to eight complexes occur in a 6-second strip D. The ST segment is higher than the PR interval E. The QRS complex ranges from 0.12 to 0.2 seconds

A,B (1) The consistency of the RR interval indicates a regular rhythm. (2) A normal P wave before each complex indicates the impulse originated in the SA node. (3) The number of complexes in a 6-second strip is multiplied by 10 to approximate the heart rate; normal sinus rhythm is 60 to 100. (4) Elevation of the ST segment is a sign of cardiac ischemia and is unrelated to the rhythm. (5) The QRS duration should be less than 0.12 seconds; the PR interval should be 0.12 to 0.2 seconds.

What criteria should the nurse use to determine normal sinus rhythm for a client on a cardiac monitor? Check all that apply. A. The RR intervals are relatively consistent B. One P wave precedes each QRS complex C. Four to eight complexes occur in a 6 second strip D. The ST segment is higher than the PR interval E. The QRS complex ranges from 0.12 to 0.20 second

A,B The consistency of the RR interval indicates regular rhythm. A normal P wave before each complex indicates the impulse originated in the SA node. The number of complexes in a 6 second strip is multiplied by 10 to approximate the heart rate; normal sinus rhythm is 60 to 100. Elevation of the ST segment is a sign of cardiac ischemia and is unrelated to the rhythm. The QRS duration should be less than 0.12 second; the PR interval should be 0.12 to 0.20 second.

A patient who has a diagnosis of atrial fibrillation as a heart rate of 152 beats per minute. The healthcare provider should assess for which of these problems related to the dysrhythmia? A. Chest pain B. Hypotension C. Headache D. Pulse deficit E. Dizziness

A,B,D,E chest pain, hypotension, pulse deficit, dizziness Atrial kick is lost and cardiac output is diminished. The result is less blood supplying the brain (dizziness), hypotension, less blood perfusing the coronary arteries, and a pulse deficit because the stroke volume is not sufficient to produce a palpable peripheral pulse.

The nurse is preparing to teach the pt newly diagnosed with angina pectoris how to self-administer nitroglycerine (NTG) tablets. What pt teaching should the nurse emphasize? select all that apply A. stop all activities immediately and sit down if chest pain occurs B. if the pain is unrelieved after three NTG tablets, continue to rest and call the prescriber right away C. After discharge, keep NTG tablets on your bedside table day and night D. immediately after chest pain begins, notify your personal prescriber E. the NTG tablet should tingle when placed under the tongue

A,B,E

When interpreting an ECG, the nurse would keep in mind which of the following about the P wave? Select all that apply. A. Reflects electrical impulse beginning at the SA node B. Indicated electrical impulse beginning at the AV node C. Reflects atrial muscle depolarization D. Identifies ventricular muscle depolarization E. Has duration of normally 0.11 seconds or less

A,C,E In a client who has had an ECG, the P wave represents the activation of the electrical impulse in the SA node, which is then transmitted to the AV node. In addition, the P wave represents atrial muscle depolarization, not ventricular depolarization. The normal duration of the P wave is 0.11 seconds or less in duration and 2.5 mm or more in height.

To determine the effects of therapy for a patient who is being treated for heart failure, which laboratory result will the nurse plan to review? a. Myoglobin b. Homocysteine (Hcy) c. Low-density lipoprotein (LDL) d. B-type natriuretic peptide (BNP)

ANS: D Increased levels of BNP are a marker for heart failure. The other laboratory results would be used to assess for myocardial infarction (myoglobin) or risk for coronary artery disease (Hcy and LDL).

A week after experiencing a ruptured cerebral aneurysm, if a patient becomes extremely indecisive and has frequent episodes of incontinence, it is probably caused by A. a vasospasm. B. automatisms. C. focal seizures. D. early-stage dementia.

A. a vasospasm.

A physical assessment finding that the nurse would expect to be present in the patient with acute left-sided heart failure is A. bubbling crackles and tachycardia. B. hepatosplenomegaly and tachypnea. C. peripheral edema and cool, diaphoretic skin. D. frothy blood-tinged sputum and distended jugular veins.

A. bubbling crackles and tachycardia.

Four hours after the onset of pain from an MI, a nurse should expect an increase in the A. creatine kinase-MB (CK-MB). B. leukocyte count. C. alkaline phosphatase (ALP). D. lactate dehydrogenase (LHD).

A. creatine kinase-MB (CK-MB).

A physician writes the following admitting orders for a patient who has fever and chills and in whom infective endocarditis is suspected: ceftriaxone (Rocephin) 1.0 g IVPB every 12 hours, aspirin for temperature above 102° F (38.9° C), and blood cultures in triplicate, complete blood count, serum chemistries, and ECG. When admitting the patient, the nurse should give the highest priority to A. obtaining the blood cultures. B. scheduling the ECG. C. administering the antipyretic agent. D. initiating the IV antibiotic.

A. obtaining the blood cultures.

A patient with no history of heart disease is seen in the clinic for periodic episodes of tachycardia with a regular rhythm. When obtaining the patient's history, the nurse should question the patient regarding the incidence of A. stress. B. asthma. C. diabetes. D. weight gain.

A. stress.

When assessing a patient with arterial insufficiency, the nurse should expect A. tissue atrophy. B. thin, fragile toenails. C. warm, erythematous legs. D. bounding arterial pulses

A. tissue atrophy.

When assessing a newly admitted patient, the nurse notes a thrill along the left sternal border. To obtain more information about the cause of the thrill, which action will the nurse take next? a. Auscultate for any cardiac murmurs. b. Find the point of maximal impulse. c. Compare the apical and radial pulse rates. d. Palpate the quality of the peripheral pulses.

ANS: A Both thrills and murmurs are caused by turbulent blood flow, such as occurs when blood flows through a damaged valve. Relevant information includes the quality of the murmur, where in the cardiac cycle the murmur is heard, and where on the thorax the murmur is heard best. The other information also is important in the cardiac assessment but will not provide information that is relevant to the thrill.

Which information about a patient who has been receiving fibrinolytic therapy for an acute myocardial infarction (AMI) is most important for the nurse to communicate to the health care provider? a. No change in the patient's chest pain b. A large bruise at the patient's IV insertion site c. A decrease in ST segment elevation on the electrocardiogram (ECG) d. An increase in cardiac enzyme levels since admission

ANS: A Continued chest pain suggests that the fibrinolytic therapy is not effective and that other interventions such as percutaneous coronary intervention (PCI) may be needed. Bruising is a possible side effect of fibrinolytic therapy, but it is not an indication that therapy should be discontinued. The decrease of the ST segment elevation indicates that fibrinolysis is occurring and perfusion is returning to the injured myocardium. An increase in cardiac enzyme levels is expected with reperfusion and is related to the washout of enzymes into the circulation as the blocked vessel is opened.

A patient who has recently started taking rosuvastatin (Crestor) and niacin (Nicobid) reports all the following symptoms to the nurse. Which is most important to communicate to the health care provider? a. Generalized muscle aches and pains b. Skin flushing after taking the medications c. Dizziness when changing positions quickly d. Nausea when taking the drugs before eating

ANS: A Muscle aches and pains may indicate myopathy and rhabdomyolysis, which have caused acute renal failure and death in some patients who have taken the statin medications. These symptoms indicate that the rosuvastatin may need to be discontinued. The other symptoms are common side effects when taking niacin, and although the nurse should follow up with the patient, they do not indicate that a change in medication is needed.

After the nurse teaches the patient about the use of atenolol (Tenormin) in preventing anginal episodes, which statement by a patient indicates that the teaching has been effective? a. "It is important not to suddenly stop taking the atenolol." b. "Atenolol will increase the strength of my heart muscle." c. "I can expect to feel short of breath when taking atenolol." d. "Atenolol will improve the blood flow to my coronary arteries."

ANS: A Patients who have been taking -blockers can develop intense and frequent angina if the medication is suddenly discontinued. Atenolol (Tenormin) decreases myocardial contractility. Shortness of breath that occurs when taking -blockers for angina may be due to bronchospasm and should be reported to the health care provider. Atenolol works by decreasing myocardial oxygen demand, not by increasing blood flow to the coronary arteries.

After a healthcare provider prescribed propranolol for a pt with frequent premature ventricular contractions, the nurse should include with of the following in the care plan? A. inform pt that excessive respiratory response to activity should gradually decrease B. measure heart rate daily before taking dose C. Pt will have increased resistance to infection D. current skin eruptions will improve within 30 days

B

The nurse hears a murmur between the S1 and S2 heart sounds at the patient's left 5th intercostal space and midclavicular line. How will the nurse record this information? a. "Systolic murmur heard at mitral area." b. "Diastolic murmur heard at aortic area." c. "Systolic murmur heard at Erb's point." d. "Diastolic murmur heard at tricuspid area."

ANS: A The S1 signifies the onset of ventricular systole. S2 signifies the onset of diastole. A murmur occurring between these two sounds is a systolic murmur. The mitral area is the intersection of the left 5th intercostal space and the midclavicular line. The other responses describe murmurs heard at different landmarks on the chest and/or during the diastolic phase of the cardiac cycle.

A patient who is being admitted to the emergency department with severe chest pain gives the following list of medications taken at home to the nurse. Which of the medications has the most immediate implications for the patient's care? a. sildenafil (Viagra) b. furosemide (Lasix) c. diazepam (Valium) d. captopril (Capoten)

ANS: A The nurse will need to avoid giving nitrates to the patient because nitrate administration is contraindicated in patients who are using sildenafil because of the risk of sudden death caused by vasodilation. The other home medications also should be documented and reported to the health care provider but do not have as immediate an impact on decisions about the patient's treatment.

Four days after having a myocardial infarction (MI), a patient who is scheduled for discharge asks for assistance with all the daily activities, saying, "I am too nervous to take care of myself." Based on this information, which nursing diagnosis is appropriate? a. Ineffective coping related to anxiety b. Activity intolerance related to weakness c. Denial related to lack of acceptance of the MI d. Social isolation related to lack of support system

ANS: A The patient data indicates that ineffective coping after the MI caused by anxiety about the impact of the MI is a concern. The other nursing diagnoses may be appropriate for some patients after an MI, but the data for this patient do not support denial, activity intolerance, or social isolation.

A patient who has chest pain is admitted to the emergency department (ED), and all the following diagnostic tests are ordered. Which one will the nurse arrange to be completed first? a. Electrocardiogram (ECG) b. Computed tomography (CT) scan c. Chest x-ray d. Troponin level

ANS: A The priority for the patient is to determine whether an acute myocardial infarction (AMI) is occurring so that reperfusion therapy can begin as quickly as possible. ECG changes occur very rapidly after coronary artery occlusion. Troponin levels will increase after about 3 hours. Data from the CT scan and chest x-ray may impact the patient's care but are not helpful in determining whether the patient is experiencing a myocardial infarction (MI).

When evaluating the outcomes of preoperative teaching with a patient scheduled for a coronary artery bypass graft (CABG) using the internal mammary artery, the nurse determines that additional teaching is needed when the patient says, a. "I will have incisions in my leg where they will remove the vein." b. "They will circulate my blood with a machine during the surgery." c. "I will need to take an aspirin a day after the surgery to keep the graft open." d. "They will use an artery near my heart to bypass the area that is obstructed."

ANS: A When the internal mammary artery is used there is no need to have a saphenous vein removed from the leg. The other statements by the patient are accurate and indicate that the teaching has been effective.

When admitting a patient with a myocardial infarction (MI) to the intensive care unit, which action should the nurse carry out first? a. Obtain the blood pressure. b. Attach the cardiac monitor. c. Assess the peripheral pulses. d. Auscultate the breath sound

ANS: B Because dysrhythmias are the most common complication of MI, the first action should be to place the patient on a cardiac monitor. The other actions also are important and should be accomplished as quickly as possible.

The nurse has received the laboratory results for a patient who developed chest pain 4 hours ago and may be having a myocardial infarction. The most important laboratory result to review will be a. LDL cholesterol. b. troponins T and I. c. C-reactive protein. d. creatine kinase-MB (CK-MB).

ANS: B Cardiac troponins start to elevate hours (average 4 to 6 hours) after myocardial injury and are specific to myocardium. Creatine kinase (CK-MB) is specific to myocardial injury and infarction, but it does not increase until 6 hours after the infarction occurs. LDL cholesterol and C-reactive protein are useful in assessing cardiovascular risk but are not helpful in determining whether a patient is having an acute myocardial infarction.

Which information collected by the nurse who is admitting a patient with chest pain suggests that the pain is caused by an acute myocardial infarction (AMI)? a. The pain increases with deep breathing. b. The pain has persisted longer than 30 minutes. c. The pain worsens when the patient raises the arms. d. The pain is relieved after the patient takes nitroglycerin.

ANS: B Chest pain that lasts for 20 minutes or more is characteristic of AMI. Changes in pain that occur with raising the arms or with deep breathing are more typical of pericarditis or musculoskeletal pain. Stable angina is usually relieved when the patient takes nitroglycerin.

For a patient who has been admitted the previous day to the coronary care unit with an acute myocardial infarction (AMI), the nurse will anticipate teaching about a. typical emotional responses to AMI. b. when patient cardiac rehabilitation will begin. c. discharge drugs such as aspirin and -blockers. d. the pathophysiology of coronary artery disease.

ANS: B Early after an AMI, the patient will want to know when resumption of usual activities can be expected. At this time, the patient's anxiety level or denial will prevent good understanding of complex information such as coronary artery disease (CAD) pathophysiology. Teaching about discharge medications should be done when the time for discharge is closer. The nurse should support the patient by decreasing anxiety rather than discussing the typical emotional response to myocardial infarction (MI).

A patient with ST segment elevation in several electrocardiographic (ECG) leads is admitted to the emergency department (ED) and diagnosed as having an ST-segment-elevation myocardial infarction (STEMI). Which question should the nurse ask to determine whether the patient is a candidate for fibrinolytic therapy? a. "Do you take aspirin on a daily basis?" b. "What time did your chest pain begin?" c. "Is there any family history of heart disease?" d. "Can you describe the quality of your chest pain?"

ANS: B Fibrinolytic therapy should be started within 6 hours of the onset of the myocardial infarction (MI), so the time at which the chest pain started is a major determinant of the appropriateness of this treatment. The other information also will be needed, but it will not be a factor in the decision about fibrinolytic therapy.

When caring for a patient who has survived a sudden cardiac death (SCD) event and has no evidence of an acute myocardial infarction (AMI), the nurse will anticipate teaching the patient a. that sudden cardiac death events rarely reoccur. b. about the purpose of outpatient Holter monitoring. c. how to self-administer low-molecular-weight heparin. d. to limit activities after discharge to prevent future events.

ANS: B Holter monitoring is used to determine whether the patient is experiencing dysrhythmias such as ventricular tachycardia during normal daily activities. SCD is likely to recur. Heparin will not have any effect on the incidence of SCD, and SCD can occur even when the patient is resting.

Amlodipine (Norvasc) is ordered for a patient with newly diagnosed Prinzmetal's (variant) angina. When teaching the patient, the nurse will include the information that amlodipine will a. reduce the "fight or flight" response. b. decrease spasm of the coronary arteries. c. increase the force of myocardial contraction. d. help prevent clotting in the coronary arteries.

ANS: B Prinzmetal's angina is caused by coronary artery spasm. Calcium channel blockers (e.g., amlodipine, nifedipine [Procardia]) are a first-line therapy for this type of angina. Platelet inhibitors, such as aspirin, help prevent coronary artery thrombosis, and -blockers decrease sympathetic stimulation of the heart. Medications or activities that increase myocardial contractility will increase the incidence of angina by increasing oxygen demand.

A patient who has had an acute myocardial infarction (AMI) asks the nurse about when sexual intercourse can be resumed. Which response by the nurse is best? a. "Most patients are able to enjoy intercourse without any complications." b. "Sexual activity uses about as much energy as climbing two flights of stairs." c. "The doctor will discuss sexual intercourse when your heart is strong enough." d. "Holding and cuddling are good ways to maintain intimacy after a heart attack."

ANS: B Sexual activity places about as much physical stress on the cardiovascular system as climbing two flights of stairs. The other responses do not directly address the patient's question, or may not be accurate for this patient.

When admitting a patient for a coronary arteriogram and angiogram, which information about the patient is most important for the nurse to communicate to the health care provider? a. The patient's pedal pulses are +1. b. The patient is allergic to shellfish. c. The patient has not eaten anything today. d. The patient had an arteriogram a year ago.

ANS: B The contrast dye used for the procedure is iodine based, so patients who have shellfish allergies will require treatment with medications such as corticosteroids and antihistamines before the arteriogram. The other information also is communicated to the health care provider but will not require a change in the usual prearteriogram orders or medications.

Which action will the nurse implement for a patient who arrives for a calcium-scoring CT scan? a. Administer oral sedative medications. b. Teach the patient about the procedure. c. Ask whether the patient has eaten today. d. Insert a large gauge intravenous catheter.

ANS: B The nurse will need to teach the patient that the procedure is rapid and involves little risk. The other actions are not necessary.

All potassium-sparing diuretics: A. are required during blood transfusions B. enhance aldosterone action C. cause hypokalemia D. are weak diuretics

B

As a competent nurse, you are aware that vasodilators are used mainly to treat: A. diabetes B. hypertension C. atrial fibrillation D. hypotension

B

A few days after experiencing a myocardial infarction (MI), the patient states, "I just had a little chest pain. As soon as I get out of here, I'm going for my vacation as planned." Which response should the nurse make? a. "Where are you planning to go for your vacation?" b. "What do you think caused your chest pain episode?" c. "Sometimes plans need to change after a heart attack." d. "Recovery from a heart attack takes at least a few weeks."

ANS: B When the patient is experiencing denial, the nurse should assist the patient in testing reality until the patient has progressed beyond this step of the emotional adjustment to MI. Asking the patient about vacation plans reinforces the patient's plan, which is not appropriate in the immediate post-MI period. Reminding the patient in denial about the MI is likely to make the patient angry and lead to distrust of the nursing staff.

Following an acute myocardial infarction (AMI), a patient ambulates in the hospital hallway. When the nurse is evaluating the patient's response, which of these assessment data would indicate that the exercise level should be decreased? a. BP changes from 118/60 to 126/68 mm Hg. b. Oxygen saturation drops from 100% to 98%. c. Heart rate increases from 66 to 90 beats/minute. d. Respiratory rate goes from 14 to 22 breaths/minute.

ANS: C A change in heart rate of more than 20 beats or more indicates that the patient should stop and rest. The increases in BP and respiratory rate, and the slight decrease in oxygen saturation, are normal responses to exercise.

A patient is scheduled for a cardiac catheterization with coronary angiography. Before the test, the nurse informs the patient that a. electrocardiographic (ECG) monitoring will be required for 24 hours after the test. b. it will be important to lie completely still during the procedure. c. a warm feeling may be noted when the contrast dye is injected. d. monitored anesthesia care will be provided during the procedure.

ANS: C A sensation of warmth or flushing is common when the iodine-based contrast material is injected, which can be anxiety-producing unless it has been discussed with the patient. The patient may receive a sedative drug before the procedure, but monitored anesthesia care is not used. ECG monitoring is used during the procedure to detect dysrhythmias, but there is not a risk for dysrhythmias after the procedure. The patient is not immobile during cardiac catheterization and may be asked to cough or take deep breaths.

A patient admitted to the coronary care unit (CCU) with an ST-segment-elevation myocardial infarction (STEMI) is restless and anxious. The blood pressure is 86/40 and heart rate is 110. Based on this information, which nursing diagnosis is a priority for the patient? a. Acute pain related to myocardial ischemia b. Anxiety related to perceived threat of death c. Decreased cardiac output related to cardiogenic shock d. Activity intolerance related to decreased cardiac output

ANS: C All the nursing diagnoses may be appropriate for this patient, but the hypotension indicates that the priority diagnosis is decreased cardiac output, which will decrease perfusion to all vital organs (e.g., brain, kidney, heart).

Which of these statements made by a patient with coronary artery disease after the nurse has completed teaching about the therapeutic lifestyle changes (TLC) diet indicates that further teaching is needed? a. "I will switch from whole milk to 1% or nonfat milk." b. "I like fresh salmon and I will plan to eat it more often." c. "I will miss being able to eat peanut butter sandwiches." d. "I can have a cup of coffee with breakfast if I want one."

ANS: C Although only 30% of the daily calories should come from fats, most of the fat in the TLC diet should come from monosaturated fats such as are found in nuts, olive oil, and canola oil. The patient can include peanut butter sandwiches as part of the TLC diet. The other patient comments indicate a good understanding of the TLC diet.

To assist the patient with coronary artery disease (CAD) in making appropriate dietary changes, which of these nursing interventions will be most effective? a. Instruct the patient that a diet containing no saturated fat and minimal sodium will be necessary. b. Emphasize the increased risk for cardiac problems unless the patient makes the dietary changes. c. Assist the patient to modify favorite high-fat recipes by using monosaturated oils when possible. d. Provide the patient with a list of low-sodium, low-cholesterol foods that should be included in the diet.

ANS: C Lifestyle changes are more likely to be successful when consideration is given to the patient's values and preferences. The highest percentage of calories from fat should come from monosaturated fats. Although low-sodium and low-cholesterol foods are appropriate, providing the patient with a list alone is not likely to be successful in making dietary changes. Removing saturated fat from the diet completely is not a realistic expectation; up to 7% of calories in the therapeutic lifestyle changes (TLC) diet can come from saturated fat. Telling the patient about the increased risk without assisting further with strategies for dietary change is unlikely to be successful.

Blurred vision or halos are signs of: A. beta blocker toxicity B. digoxin toxicity C. diuretic toxicity D. ACE inhibitor toxicity

B

Which information obtained by the nurse who is admitting the patient for magnetic resonance imaging (MRI) will be most important to report to the health care provider before the MRI? a. The patient has an allergy to shellfish and iodine. b. The patient has a history of coronary artery disease. c. The patient has a permanent ventricular pacemaker in place. d. The patient took all the prescribed cardiac medications today.

ANS: C MRI is contraindicated for patients with implanted metallic devices such as pacemakers. The other information also will be reported to the health care provider but does not impact on whether or not the patient can have an MRI.

During the administration of the fibrinolytic agent to a patient with an acute myocardial infarction (AMI), the nurse should stop the drug infusion if the patient experiences a. bleeding from the gums. b. surface bleeding from the IV site. c. a decrease in level of consciousness. d. a nonsustained episode of ventricular tachycardia.

ANS: C The change in level of consciousness indicates that the patient may be experiencing intracranial bleeding, a possible complication of fibrinolytic therapy. Bleeding of the gums and prolonged bleeding from IV sites are expected side effects of the therapy. The nurse should address these by avoiding any further injuries, but they are not an indication to stop infusion of the fibrinolytic medication. A nonsustained episode of ventricular tachycardia is a common reperfusion dysrhythmia and may indicate that the therapy is effective.

Which assessment finding by the nurse who is caring for a patient who has had coronary artery bypass grafting using a right radial artery graft is most important to communicate to the physician? a. Complaints of incisional chest pain b. Crackles audible at both lung bases c. Pallor and weakness of the right hand d. Redness on either side of the chest incision

ANS: C The changes in the right hand indicate compromised blood flow, which requires immediate evaluation and actions such as prescribed calcium channel blockers or surgery. The other changes are expected and/or require nursing interventions

After the nurse has finished teaching a patient about use of sublingual nitroglycerin (Nitrostat), which patient statement indicates that the teaching has been effective? a. "I can expect indigestion as a side effect of nitroglycerin." b. "I can only take the nitroglycerin if I start to have chest pain." c. "I will call an ambulance if I still have pain 5 minutes after taking the nitroglycerin." d. "I will help slow down the progress of the plaque formation by taking nitroglycerin."

ANS: C The emergency medical services (EMS) system should be activated when chest pain or other symptoms are not completely relieved 5 minutes after taking one nitroglycerin. Nitroglycerin can be taken to prevent chest pain or other symptoms from developing (e.g., before intercourse). Gastric upset is not an expected side effect of nitroglycerin. Nitroglycerin does not impact the underlying pathophysiology of coronary artery atherosclerosis.

The nurse teaches the patient being evaluated for rhythm disturbances with a Holter monitor to a. exercise more than usual while the monitor is in place. b. remove the electrodes when taking a shower or tub bath. c. keep a diary of daily activities while the monitor is worn. d. connect the recorder to a telephone transmitter once daily.

ANS: C The patient is instructed to keep a diary describing daily activities while Holter monitoring is being accomplished to help correlate any rhythm disturbances with patient activities. Patients are taught that they should not take a shower or bath during Holter monitoring and that they should continue with their usual daily activities. The recorder stores the information about the patient's rhythm until the end of the testing, when it is removed and the data are analyzed.

A transesophageal echocardiogram (TEE) is ordered for a patient with possible endocarditis. Which of these actions included in the standard TEE orders will the nurse need to accomplish first? a. Administer O2 per mask. b. Start a large-gauge IV line. c. Place the patient on NPO status. d. Give lorazepam (Ativan) 1 mg IV.

ANS: C The patient will need to be NPO for 6 hours preceding the TEE, so the nurse should place the patient on NPO status as soon as the order is received. The other actions also will need to be accomplished but not until just before or during the procedure

Three days after a myocardial infarction (MI), the patient develops chest pain that increases when taking a deep breath and is relieved by leaning forward. Which action should the nurse take next? a. Palpate the radial pulses bilaterally. b. Assess the feet for peripheral edema. c. Auscultate for a pericardial friction rub. d. Check the cardiac monitor for dysrhythmias.

ANS: C The patient's symptoms are consistent with the development of pericarditis, a possible complication of MI. The other assessments listed are not consistent with the description of the patient's symptoms.

The nurse has just received change-of-shift report about the following four patients. Which patient should the nurse assess first? a. 38-year-old who has pericarditis and is complaining of sharp, stabbing chest pain b. 45-year-old who had a myocardial infarction (MI) 4 days ago and is anxious about the planned discharge c. 51-year-old with unstable angina who has just returned to the unit after having a percutaneous coronary intervention (PCI) d. 60-year-old with variant angina who is to receive a scheduled dose of nifedipine (Procardia)

ANS: C This patient is at risk for bleeding from the arterial access site for the PCI, so the nurse should assess the patient's blood pressure, pulse, and the access site immediately. The other patients also should be assessed as quickly as possible, but assessment of this patient has the highest priority.

When caring for a patient who has just arrived on the medical-surgical unit after having cardiac catheterization, which nursing action should the nurse delegate to an LPN/LVN? a. Perform the initial assessment of the catheter insertion site. b. Teach the patient about the usual postprocedure plan of care. c. Check the rate on the infusion pump used to administer heparin. d. Administer the scheduled aspirin and lipid-lowering medication.

ANS: D Administration of oral medications is within the scope of practice for LPNs/LVNs. The initial assessment of the patient, patient teaching, and administration of intravenous anticoagulant medications should be done by the RN.

When developing a health teaching plan for a 60-year-old man with the following risk factors for coronary artery disease (CAD), the nurse should focus on the a. family history of coronary artery disease. b. increased risk associated with the patient's gender. c. high incidence of cardiovascular disease in older people. d. elevation of the patient's serum low density lipoprotein (LDL) level.

ANS: D Because family history, gender, and age are nonmodifiable risk factors, the nurse should focus on the patient's LDL level. Decreases in LDL will help reduce the patient's risk for developing CAD.

Nadolol (Corgard) is prescribed for a patient with angina. To determine whether the drug is effective, the nurse will monitor for a. decreased blood pressure and apical pulse rate. b. fewer complaints of having cold hands and feet. c. improvement in the quality of the peripheral pulses. d. the ability to do daily activities without chest discomfort.

ANS: D Because the medication is ordered to improve the patient's angina, effectiveness is indicated if the patient is able to accomplish daily activities without chest pain. Blood pressure (BP) and apical pulse rate may decrease, but these data do not indicate that the goal of decreased angina has been met. The noncardioselective -blockers can cause peripheral vasoconstriction, so the nurse would not expect an improvement in peripheral pulse quality or skin temperature

Which information given by a patient admitted with chronic stable angina will help the nurse confirm this diagnosis? a. The patient rates the pain at a level 3 to 5 (0 to 10 scale). b. The patient states that the pain "wakes me up at night." c. The patient says that the frequency of the pain has increased over the last few weeks. d. The patient states that the pain is resolved after taking one sublingual nitroglycerin tablet.

ANS: D Chronic stable angina is typically relieved by rest or nitroglycerin administration. The level of pain is not a consistent indicator of the type of angina. Pain occurring at rest or with increased frequency is typical of unstable angina.

When the nurse is monitoring a patient who is undergoing exercise (stress) testing on a treadmill, which assessment finding requires the most rapid action by the nurse? a. Patient complaint of feeling tired. b. Pulse change from 80 to 96 beats/minute. c. BP increase from 134/68 to 150/80 mm Hg. d. Electrocardiographic (ECG) changes indicating coronary ischemia.

ANS: D ECG changes associated with coronary ischemia (such as T-wave inversions and ST segment depression) indicate that the myocardium is not getting adequate oxygen delivery and that the exercise test should be terminated immediately. Increases in BP and heart rate (HR) are normal responses to aerobic exercise. Tiredness also is normal as the intensity of exercise increases during the stress testing.

A patient with a non-ST-segment-elevation myocardial infarction (NSTEMI) is receiving heparin. What is the purpose of the heparin? a. Platelet aggregation is enhanced by IV heparin infusion. b. Heparin will dissolve the clot that is blocking blood flow to the heart. c. Coronary artery plaque size and adherence are decreased with heparin. d. Heparin will prevent the development of new clots in the coronary arteries

ANS: D Heparin helps prevent the conversion of fibrinogen to fibrin and decreases coronary artery thrombosis. It does not change coronary artery plaque, dissolve already formed clots, or enhance platelet aggregation.

Which of these nursing interventions included in the plan of care for a patient who had an acute myocardial infarction (AMI) 3 days ago is most appropriate for the RN to delegate to an experienced LPN/LVN? a. Evaluating the patient's response to ambulation in the hallway b. Completing the documentation for a home health nurse referral c. Educating the patient about the pathophysiology of heart disease d. Reinforcing teaching about the purpose of prescribed medications

ANS: D LPN/LVN education and scope of practice include reinforcing education that has previously been done by the RN. Evaluating the patient response to exercise after an AMI requires more education and should be done by the RN. Teaching and discharge planning/documentation are higher level skills that require RN education and scope of practice.

The nurse will suspect that the patient with stable angina is experiencing a side effect of the prescribed metoprolol (Lopressor) if a. the patient is restless and agitated. b. the blood pressure is 190/110 mm Hg. c. the patient complains about feeling anxious. d. the cardiac monitor shows a heart rate of 45.

ANS: D Patients taking -blockers should be monitored for bradycardia. Because this category of medication inhibits the sympathetic nervous system, restlessness, agitation, hypertension, and anxiety will not be side effects.

A patient with hyperlipidemia has a new order for colesevelam (Welchol). Which nursing action is appropriate when giving the medication? a. Administer the medication at the patient's bedtime. b. Have the patient take this medication with an aspirin. c. Encourage the patient to take the colesevelam with a sip of water. d. Give the patient's other medications 2 hours after the colesevelam.

ANS: D The bile acid sequestrants interfere with the absorption of other drugs, and giving other medications at the same time should be avoided. Taking an aspirin concurrently with the colesevelam may increase the incidence of gastrointestinal side effects such as heartburn. An increased fluid intake is encouraged for patients taking the bile acid sequestrants to reduce the risk for constipation. For maximum effect, colesevelam should be administered with meals.

The nurse obtains the following data when caring for a patient who experienced an acute myocardial infarction (AMI) 2 days previously. Which information is most important to report to the health care provider? a. The patient denies ever having a heart attack. b. The cardiac-specific troponin level is elevated. c. The patient has occasional premature atrial contractions (PACs). d. Crackles are auscultated bilaterally in the mid-lower lobes.

ANS: D The crackles indicate that the patient may be developing heart failure, a possible complication of myocardial infarction (MI). The health care provider may need to order medications such as diuretics or angiotensin-converting enzyme (ACE) inhibitors for the patient. Elevation in cardiac troponin level at this time is expected. PACs are not life-threatening dysrhythmias. Denial is a common response in the immediate period after the MI.

When administering IV nitroglycerin (Tridil) to a patient with a myocardial infarction (MI), which action will the nurse take to evaluate the effectiveness of the medication? a. Check blood pressure. b. Monitor apical pulse rate. c. Monitor for dysrhythmias. d. Ask about chest discomfort.

ANS: D The goal of IV nitroglycerin administration in MI is relief of chest pain by improving the balance between myocardial oxygen supply and demand. The nurse also will monitor heart rate and BP and observe for dysrhythmias, but these parameters will not indicate whether the medication is effective.

Which electrocardiographic (ECG) change is most important for the nurse to communicate to the health care provider when caring for a patient with chest pain? a. Frequent premature atrial contractions (PACs) b. Inverted P wave c. Sinus tachycardia d. ST segment elevation

ANS: D The patient is likely to be experiencing an ST-segment-elevation myocardial infarction (STEMI) and immediate therapy with percutaneous coronary intervention (PCI) or fibrinolytic medications is indicated to minimize the amount of myocardial damage. The other ECG changes also may suggest a need for therapy, but not as rapidly.

After the nurse teaches a patient with chronic stable angina about how to use the prescribed short-acting and long-acting nitrates, which statement by the patient indicates that the teaching has been effective? a. "I will put on the nitroglycerin patch as soon as I develop any chest pain." b. "I will check the pulse rate in my wrist just before I take any nitroglycerin." c. "I will be sure to remove the nitroglycerin patch before using any sublingual nitroglycerin." d. "I will stop what I am doing and sit down before I put the nitroglycerin under my tongue."

ANS: D The patient should sit down before taking the nitroglycerin to decrease cardiac workload and prevent orthostatic hypotension. Transdermal nitrates are used prophylactically rather than to treat acute pain and can be used concurrently with sublingual nitroglycerin. Although the nurse should check blood pressure before giving nitroglycerin, patients do not need to check the pulse rate before taking nitrates.

When caring for a patient with acute coronary syndrome who has returned to the coronary care unit after having balloon angioplasty, the nurse obtains the following assessment data. Which data indicate the need for immediate intervention by the nurse? a. Pedal pulses 1+ b. Heart rate 100 beats/min c. Blood pressure 104/56 mm Hg d. Chest pain level 8 on a 10-point scale

ANS: D The patient's chest pain indicates that restenosis of the coronary artery may be occurring and requires immediate actions, such as administration of oxygen and nitroglycerin, by the nurse. The other information indicates a need for ongoing assessments by the nurse.

Mira is managing her hypertension with an ACE inhibitor. Which of the following statements stated by her indicates a need for further teaching? A. I should not take my pills with food B. I need to increase my intake of orange juice, bananas, and green veggies C. I will avoid coffee, tea, and cola D. I will avoid salt substitutes

B

SC heparin should be administered in the: A. flank B. abdominal fat C. thigh D. buttock

B

The healthcare provider is performing an assessment on a patient who is taking propranolol (Inderal) for supraventricular tachycardia. Which assessment finding is an indication the patient is experiencing an adverse effect of this drug? Please choose from one of the following options. A. Dry mouth B. Bradycardia C. Urinary retention D. Paresthesia

B

The nurse feels pulsations on a client's right sternal border, second space. What does this finding suggest to the nurse? a. A prolapsed mitral valve b. Aortic stenosis c. Nothing. This is a normal finding. d. Tricuspid valve regurgitation

B

The nurse is monitoring a pt taking digoxin (Lanoxin) for treatment of heart failure. Which assessment finding indicates a therapeutic effect of the drug? A. HR 110 beats/minute B. HR 58 beats/minute C. urinary output 40 mL/hr D. BP 90/50 mm Hg

B

When a pt is experiencing digitalis toxicity, in which of the following situations would it be appropriate to treat with digoxin immune Fab (Digibind)? A. hypokalemia B. hyperkalemia C. apical heart rate of 60 bpm D. supraventricular dysrhythmias

B

When teaching the pt about the signs and symptoms of cardiac glycoside toxicity, the nurse should alert the pt to watch for: A. visual changes B. flickering lights or halos C. dizziness when standing up D. increased urine output

B

When ventricular fibrillation occurs in a CCU, the first person reaching the client should: A. Administer oxygen B. Defibrillate the client C. Initiate CPR D. Administer sodium bicarbonate intravenously

B

Which of the following adverse effects is of most concern for the older adult pt taking anti-hypertensive drugs A. dry mouth B. hypotension C. restlessness D. constipation

B

Your pt is currently taking Digoxin. What should you, as a nurse be prepared to administer in the event of digitalis toxicity? A. potassium B. digibind C. protamine sulfate D. heparin

B

A 56 y/o man started antihypertensive drug therapy 3 months earlier and is in the office for a follow-up visit. While the nurse is taking his BP, he informs the nurse that he has had some problems with sexual intercourse. Which of the following would be the most appropriate response by the nurse: A. not to worry. Tolerance will develop B. the physician can work with you on changing the dose or drug C. sexual dysfunction happens with this therapy, and you must lean to accept it D. this is an unusual occurrence, but it is important to stay on your meds

B

A client comes to the outpatient clinic and tells the nurse that he has had legs pains that began when he walks but cease when he stops walking. Which of the following conditions would the nurse assess for? A. An acute obstruction in the vessels of the legs B. Peripheral vascular problems in both legs C. Diabetes D. Calcium deficiency

B

A home health nurse instructs a pt about the use of a nitrate patch. The nurse tells the pt that which of the following will prevent pt tolerance to nitrates? A. do not remove the patches B. have a 12-hr no-nitrate time C. have a 24-hr no-nitrate time D. keep nitrates on 48 hrs, then off 24 hrs

B

A patient is being discharged after the insertion of a permanent pacemaker. Which statement made by the patient indicates an understanding regarding appropriate self-care? Please choose from one of the following options. A. "Every morning I will perform arm and shoulder stretches." B. "Each day I'll take my pulse and record it in a log." C. "I'll have to get rid of my microwave oven." D. "I won't be able to use my electric blanket anymore."

B

A pt with a rapid, irregular heart rhythm is being treated in the ER with adenosine. During administration of this drug, the nurse should be prepared to monitor the pt for which effect? A. nausea and vomiting B. transitory asystole C. muscle tetany D. hypertension

B

A pt with atrial fibrillation is receiving continuous heparin infusion at 1000 units/hr. The nurse would determine that the pt is receiving the therapeutic effect based on which of the following results? A. prothrombin time of 12.5 seconds B. apt time of 60 seconds C. apt time of 28 seconds D. apt time longer than 120 sec

B

a pt is receiving thrombolytic therapy with a continuous infusion of streptokinase. The pt suddenly becomes extremely anxious with complaints of itching. A nurse hears stridor and on examination of the pt notes generalized urticarial and hypotension. Which of the following should be the priority action? A. administer oxygen and protamine sulfate B. stop the infusion and call the physician C. cut the infusion rate in half and sit the pt up in bed D. administer benadryl and continue the infusion slowly

B

the nurse reviews lab studies of a pt receiving digoxin (Lanoxin). Intervention by the nurse is required if the results include which of the following laboratory values? A. serum digoxin level of 1.2 ng/dL B. Serum potassium level of 3 mEq/L C. hemoglobin of 14.4 g/ dL D. serum sodium level of 140 mEq/L

B

The physician refers the client with unstable angina for a cardiac catheterization. The nurse explains to the client that this procedure is being used in this specific case to: A. Open and dilate the blocked coronary arteries B. Assess the extent of arterial blockage C. Bypass obstructed vessels D. Assess the functional adequacy of the valves and heart muscle

B Cardiac catheterization is done in clients with angina primarily to assess the extent and severity of the coronary artery blockage, A decision about medical management, angioplasty, or coronary artery bypass surgery will be based on the catheterization results.

A client with myocardial infarction has been transferred from a coronary care unit to a general medical unit with cardiac monitoring via telemetry. A nurse plans to allow for which of the following client activities? A. Strict bed rest for 24 hours after transfer B. Bathroom privileges and self-care activities C. Unsupervised hallway ambulation with distances under 200 feet D. Ad lib activities because the client is monitored

B On transfer from the CCU, the client is allowed self-care activities and bathroom privileges. Supervised ambulation for brief distances are encouraged, with distances gradually increased (50, 100, 200 feet).

When teaching a patient why spironolactone (Aldactone) and furosemide (Lasix) are prescribed together, the nurse bases teaching on the knowledge that: A. Moderate doses of two different types of diuretics are more effective than a large dose of one type B. This combination promotes diuresis but decreases the risk of hypokalemia C. This combination prevents dehydration and hypovolemia D. Using two drugs increases osmolality of plasma and the glomerular filtration rate

B Spironolactone is a potassium-sparing diuretic; furosemide is a potassium-losing diuretic. Giving these together minimizes electrolyte imbalance.

What is the first intervention for a client experiencing myocardial infarction? A. Administer morphine B. Administer oxygen C. Administer sublingual nitroglycerin D. Obtain an electrocardiogram

B Administering supplemental oxygen to the client is the first priority of care. The myocardium is deprived of oxygen during an infarction, so additional oxygen is administered to assist in oxygenation and prevent further damage. Morphine and sublingual nitroglycerin are also used to treat MI, but they're more commonly administered after the oxygen. An ECG is the most common diagnostic tool used to evaluate MI.

When do coronary arteries primarily receive blood flow? A. During inspiration b. During diastole C. During expiration D. During systole

B Although the coronary arteries may receive a minute portion of blood during systole, most of the blood flow to coronary arteries is supplied during diastole. Breathing patterns are irrelevant to blood flow

What is the definitive test used to diagnose an abdominal aortic aneurysm? A. Abdominal X-ray B. Arteriogram C. CT scan D. Ultrasound

B An arteriogram accurately and directly depicts the vasculature; therefore, it clearly delineates the vessels and any abnormalities. An abdominal aneurysm would only be visible on an X-ray if it were calcified. CT scan and ultrasound don't give a direct view of the vessels and don't yield as accurate a diagnosis as the arteriogram.

Atherosclerosis impedes coronary blood flow by which of the following mechanisms? A. Plaques obstruct the vein B. Plaques obstruct the artery C. Blood clots form outside the vessel wall D. Hardened vessels dilate to allow the blood to flow through

B Arteries, not veins, supply the coronary arteries with oxygen and other nutrients. Atherosclerosis is a direct result of plaque formation in the artery. Hardened vessels can't dilate properly and, therefore, constrict blood flow.

A nurse is watching the cardiac monitor, and a client's rhythm suddenly changes. There are no P waves; instead there are wavy lines. The QRS complexes measure 0.08 second, but they are irregular, with a rate of 120 beats a minute. The nurse interprets this rhythm as: A. Sinus tachycardia B. Atrial fibrillation C. Ventricular tachycardia D. Ventricular fibrillation

B Atrial fibrillation is characterized by a loss of P waves; an undulating, wavy baseline; QRS duration that is often within normal limits; and an irregular ventricular rate, which can range from 60 to 100 beats per minute (when controlled with medications) to 100 to 160 beats per minute (when uncontrolled).

Which of the following classes of drugs is most widely used in the treatment of cardiomyopathy? A. Antihypertensive B. Beta-adrenergic blockers C. Calcium channel blockers D. Nitrates

B By decreasing the heart rate and contractility, beta-adrenergic blockers improve myocardial filling and cardiac output, which are primary goals in the treatment of cardiomyopathy. Antihypertensives aren't usually indicated because they would decrease cardiac output in clients who are often already hypotensive. Calcium channel blockers are sometimes used for the same reasons as beta-adrenergic blockers; however, they aren't as effective as beta-adrenergic blockers and cause increase hypotension. Nitrates aren't' used because of their dilating effects, which would further compromise the myocardium.

Which of the following types of cardiomyopathy does not affect cardiac output? A. Dilated B. Hypertrophic C. Restrictive D. Obliterative

B Cardiac output isn't affected by hypertrophic cardiomyopathy because the size of the ventricle remains relatively unchanged. Dilated cardiomyopathy, and restrictive cardomyopathy all decrease cardiac output.

Select all that apply. Which of the following are characteristic manifestations of paroxysmal nocturnal dyspnea (PND)? A. Vomiting B. Nighttime awakening C. Migraine-type headache D. Episodes of snoring accompanied by apnea E. Relief of symptoms when the patient sits upright F. Occurrence 2 to 3 hours after the patient goes to sleep

B. Nighttime awakening E. Relief of symptoms when the patient sits upright F. Occurrence 2 to 3 hours after the patient goes to sleep

A 2-gram sodium diet is prescribed for a client with severe hypertension. The client does not like the diet, and the nurse hears the client request that the spouse "Bring in some good home-cooked food." It would be most effective for the nurse to plan to: A. Call in the dietician for client teaching B. Wait for the client's family and discuss the diet with the client and family C. Tell the client that the use of salt is forbidden, because it will raise BP D. Catch the family members before they go into the client's room and tell them about the diet.

B Clients' families should be included in dietary teaching; families provide support that promotes adherence.

Which of the following conditions is linked to more than 50% of clients with abdominal aortic aneurysms? A. DM B. HPN C. PVD D. Syphilis

B Continuous pressure on the vessel walls from hypertension causes the walls to weaken and an aneurysm to occur. Atherosclerotic changes can occur with peripheral vascular diseases and are linked to aneurysms, but the link isn't as strong as it is with hypertension. Only 1% of clients with syphilis experience an aneurysm. Diabetes mellitus doesn't have direct link to aneurysm.

A nurse is caring for a client with unstable ventricular tachycardia. The nurse instructs the client to do which of the following, if prescribed, during an episode of ventricular tachycardia? A. Breathe deeply, regularly, and easily B. Inhale deeply and cough forcefully every 1 to 3 seconds C. Lie down flat in bed D. Remove any metal jewelry

B Cough cardiopulmonary resuscitation (CPR) sometimes is used in the client with unstable ventricular tachycardia. The nurse tells the client to use cough CPR, if prescribed, by inhaling deeply and coughing forcefully every 1 to 3 seconds. Cough CPR may terminate the dysrhythmia or sustain the cerebral and coronary circulation for a short time until other measures can be implemented.

With which of the following disorders is jugular vein distention most prominent? A. Abdominal aortic aneurysm B. Heart failure C. Myocardial infarction D. Pneumothorax

B Elevated venous pressure, exhibited as jugular vein distention, indicates a failure of the heart to pump. Jugular vein distention isn't a symptom of abdominal aortic aneurysm or pneumothorax. An MI, if severe enough, can progress to heart failure; however, in and of itself, an MI doesn't cause jugular vein distention.

A client has been admitted to the hospital with a diagnosis of suspected bacterial endocarditis. The complication the nurse will constantly observe for is: A. Presence of heart murmur B. Systemic emboli C. Fever D. Congestive heart failure

B Emboli are the major problem; those arising in the right heart chambers will terminate in the lungs and left chamber emboli may travel anywhere in the arteries. Heart murmurs, fever, and night sweats may be present, but do not indicate a problem with emboli. CHF may be a result, but this is not as dangerous an outcome as emboli

A nurse notices frequent artifact on the ECG monitor for a client whose leads are connected by cable to a console at the bedside. The nurse examines the client to determine the cause. Which of the following items is unlikely to be responsible for the artifact? A. Frequent movement of the client B. Tightly secured cable connections C. Leads applied over hairy areas D. Leads applied to the limbs

B Motion artifact, or "noise," can be caused by frequent client movement, electrode placement on limbs, and insufficient adhesion to the skin, such as placing electrodes over hairy areas of the skin. Electrode placement over bony prominences also should be avoided. Signal interference can also occur with electrode removal and cable disconnection.

While caring for a client who has sustained an MI, the nurse notes eight PVCs in one minute on the cardiac monitor. The client is receiving an IV infusion of D5W and oxygen at 2 L/minute. The nurse's first course of action should be to: A. Increase the IV infusion rate B. Notify the physician promptly C. Increase the oxygen concentration D. Administer a prescribed analgesic

B PVCs are often a precursor of life-threatening dysrhythmias, including ventricular tachycardia and ventricular fibrillation. An occasional PVC is not considered dangerous, but if PVCs occur at a rate greater than 5 or 6 per minute in the post MI client, the physician should be notified immediately. More than 6 PVCs per minute is considered serious and usually calls for decreasing ventricular irritability by administering medications such as lidocaine. Increasing the IV infusion rate would not decrease the number of PVCs. Increasing the oxygen concentration should not be the nurse's first course of action; rather, the nurse should notify the physician promptly. Administering a prescribed analgesic would not decrease ventricular irritability.

The nurse identifies the collaborative problem of potential complication: pulmonary edema for a patient in ADHF. When assessing the patient, the nurse will be most concerned about a. an apical pulse rate of 106 beats/min. b. an oxygen saturation of 88% on room air. c. weight gain of 1 kg (2.2 lb) over 24 hours. d. decreased hourly patient urinary output.

B Rationale: A decrease in oxygen saturation to less than 92% indicates hypoxemia. The nurse should administer supplemental oxygen immediately to the patient. An increase in apical pulse rate, 1-kg weight gain, and decreases in urine output also indicate worsening heart failure and require rapid nursing actions, but the low oxygen saturation rate requires the most immediate nursing action.

A patient who is receiving dobutamine (Dobutrex) for the treatment of ADHF has all of the following nursing actions included in the plan of care. Which action will be best for the RN to delegate to an experienced LPN/LVN? a. Teach the patient the reasons for remaining on bed rest. b. Monitor the patient's BP every hour. c. Adjust the drip rate to keep the systolic BP >90 mm Hg. d. Call the health care provider about a decrease in urine output.

B Rationale: An experienced LPN/LVN would be able to monitor BP and would know to report significant changes to the RN. Teaching patients and making adjustments to the drip rate for vasoactive medications are RN-level skills. Because the health care provider may order changes in therapy based on the decrease in urine output, the RN should call the health care provider about the decreased urine output.

Which of the following landmarks is the correct one for obtaining an apical pulse? A. Left intercostal space, midaxillary line B. Left fifth intercostal space, midclavicular line C. Left second intercostal space, midclavicular line D. Left seventh intercostal space, midclavicular line

B The correct landmark for obtaining an apical pulse is the left intercostal space in the midclavicular line. This is the point of maximum impulse and the location of the left ventricular apex. The left second intercostal space in the midclavicular line is where the pulmonic sounds are auscultated. Normally, heart sounds aren't heard in the midaxillary line or the seventh intercostal space in the midclavicular line.

When the nurse is developing a teaching plan to prevent the development of heart failure in a patient with stage 1 hypertension, the information that is most likely to improve compliance with antihypertensive therapy is that a. hypertensive crisis may lead to development of acute heart failure in some patients. b. hypertension eventually will lead to heart failure by overworking the heart muscle. c. high BP increases risk for rheumatic heart disease. d. high systemic pressure precipitates papillary muscle rupture.

B Rationale: Hypertension is a primary cause of heart failure because the increase in ventricular afterload leads to ventricular hypertrophy and dilation. Hypertensive crisis may precipitate acute heart failure is some patients, but this patient with stage 1 hypertension may not be concerned about a crisis that happens only to some patients. Hypertension does not directly cause rheumatic heart disease (which is precipitated by infection with group A -hemolytic streptococcus) or papillary muscle rupture (which is caused by myocardial infarction/necrosis of the papillary muscle).

A patient admitted to the hospital with an exacerbation of chronic heart failure tells the nurse, "I felt fine when I went to bed, but I woke up in the middle of the night feeling like I was suffocating!" The nurse can best document this assessment information as a. pulsus alternans. b. paroxysmal nocturnal dyspnea. c. two-pillow orthopnea. d. acute bilateral pleural effusion.

B Rationale: Paroxysmal nocturnal dyspnea is caused by the reabsorption of fluid from dependent body areas when the patient is sleeping and is characterized by waking up suddenly with the feeling of suffocation. Pulsus alternans is the alternation of strong and weak peripheral pulses during palpation. Orthopnea indicates that the patient is unable to lie flat because of dyspnea. Pleural effusions develop over a longer time period.

When developing a plan to decrease preload in the patient with heart failure, the nurse will include actions such as a. administering sedatives to promote rest and decrease myocardial oxygen demand. b. positioning the patient in a high-Fowler's position with the feet horizontal in the bed. c. administering oxygen per mask or nasal cannula. d. encouraging leg exercises to improve venous return.

B Rationale: Positioning the patient in a high-Fowler's position with the legs dependent will reduce preload by decreasing venous return to the right atrium. The other interventions may also be appropriate for patients with heart failure but will not help in decreasing preload.

Intravenous sodium nitroprusside (Nipride) is ordered for a patient with acute pulmonary edema. During the first hours of administration, the nurse will need to adjust the Nipride rate if the patient develops a. a drop in heart rate to 54 beats/min. b. a systolic BP <90 mm Hg. c. any symptoms indicating cyanide toxicity. d. an increased amount of ventricular ectopy.

B Rationale: Sodium nitroprusside is a potent vasodilator, and the major adverse effect is severe hypotension. After 48 hours of continuous use, cyanide toxicity is a possible (though rare) adverse effect. Reflex tachycardia (not bradycardia) is another adverse effect of this medication. Nitroprusside does not cause increased ventricular ectopy.

The nurse working in the heart failure clinic will know that teaching for a 74-year-old patient with newly diagnosed heart failure has been effective when the patient a. says that the nitroglycerin patch will be used for any chest pain that develops. b. calls when the weight increases from 124 to 130 pounds in a week. c. tells the home care nurse that furosemide (Lasix) is taken daily at bedtime. d. makes an appointment to see the doctor at least once yearly.

B Rationale: Teaching for a patient with heart failure includes information about the need to weigh daily and notify the health care provider about an increase of 3 pounds in 2 days or 5 pounds in a week. Nitroglycerin patches are used primarily to reduce preload (not to prevent chest pain) in patients with heart failure and should be used daily, not on an "as necessary" basis. Diuretics should be taken earlier in the day to avoid nocturia and sleep disturbance. Heart failure is a chronic condition that will require frequent follow-up rather than an annual health care provider examination.

While admitting an 80-year-old patient with heart failure to the medical unit, the nurse obtains the information that the patient lives alone and sometimes confuses the "water pill" with the "heart pill." The nurse makes a note that discharge planning for the patient will need to include a. transfer to a dementia care service. b. referral to a home health care agency. c. placement in a long-term-care facility. d. arrangements for around-the-clock care.

B Rationale: The data about the patient suggest that assistance in developing a system for taking medications correctly at home is needed. A home health nurse will assess the patient's home situation and help the patient to develop a method for taking the two medications as directed. There is no evidence that the patient requires services such as dementia care, long-term-care, or around-the-clock home care.

Which of the following complications is of greatest concern when caring for a preoperative abdominal aneurysm client? A. HPN B. Aneurysm rupture C. Cardiac arrythmias D. Diminished pedal pulses

B Rupture of the aneurysm is a life-threatening emergency and is of the greatest concern for the nurse caring for this type of client. Hypertension should be avoided and controlled because it can cause the weakened vessel to rupture. Diminished pedal pulses, a sign of poor circulation to the lower extremities, are associated with an aneurysm but isn't life threatening. Cardiac arrhythmias aren't directly linked to an aneurysm.

Which of the following groups of symptoms indicated a ruptured abdominal aneurysm? A. Lower back pain, increased BP, decreased RBC, increased WBC B. Severe lower back pain, decreased BP, decreased RBC, increased WBC C. Severe lower back pain, decreased BP, decreased RBC, decreased WBC D. Intermittent lower back pain, decreased BP, decreased RBC, increased WBC

B Severe lower back pain indicates an aneurysm rupture, secondary to pressure being applied within the abdominal cavity. When rupture occurs, the pain is constant because it can't be alleviated until the aneurysm is repaired. Blood pressure decreases due to the loss of blood. After the aneurysm ruptures, the vasculature is interrupted and blood volume is lost, so blood pressure wouldn't increase. For the same reason, the RBC count is decreased - not increase. The WBC count increases as cells migrate to the site of injury.

Stimulation of the sympathetic nervous system produces which of the following responses? A. Bradycardia B. Tachycardia C. Hypotension D. Decreased myocardial contractility

B Stimulation of the sympathetic nervous system causes tachycardia and increased contractility. The other symptoms listed are related to the parasympathetic nervous system, which is responsible for slowing the heart rate.

In which of the following areas is an abdominal aortic aneurysm most commonly located? A. Distal to the iliac arteries B. Distal to the renal arteries C. Adjacent to the aortic branch D. Proximal to the renal arteries

B The portion of the aorta distal to the renal arteries is more prone to an aneurysm because the vessel isn't surrounded by stable structures, unlike the proximal portion of the aorta. Distal to the iliac arteries, the vessel is again surrounded by stable vasculature, making this an uncommon site for an aneurysm. There is no area adjacent to the aortic arch, which bends into the thoracic (descending) aorta.

A nurse notes that a client with sinus rhythm has a premature ventricular contraction that falls on the T wave of the preceding beat. The client's rhythm suddenly changes to one with no P waves or definable QRS complexes. Instead there are coarse wavy lines of varying amplitude. The nurse assesses this rhythm to be: A. Ventricular tachycardia B. Ventricular fibrillation C. Atrial fibrillation D. Asystole

B Ventricular fibrillation is characterized by irregular, chaotic undulations of varying amplitudes. Ventricular fibrillation has no measurable rate and no visible P waves or QRS complexes and results from electrical chaos in the ventricles.

A nurse is watching the cardiac monitor and notices that the rhythm suddenly changes. There are no P waves, the QRS complexes are wide, and the ventricular rate is regular but over 100. The nurse determines that the client is experiencing: A. Premature ventricular contractions B. Ventricular tachycardia C. Ventricular fibrillation D. Sinus tachycardia

B Ventricular tachycardia is characterized by the absence of P waves, wide QRS complexes (usually greater than 0.14 second), and a rate between 100 and 250 impulses per minute. The rhythm is usually regular.

A nurse is caring for a patient with a diagnosis of deep venous thrombosis (DVT). The patient has an order to receive 30 mg enoxaparin (Lovenox). Which injection site should the nurse use to administer this medication safely? A) Buttock, upper outer quadrant B) Abdomen, anterior-lateral aspect C) Back of the arm, 2 inches away from a mole D) Anterolateral thigh, with no scar tissue nearby

B) Enoxaparin (Lovenox) is a low-molecular-weight (LMW) heparin that is given as a deep subcutaneous injection in the right and left anterolateral abdomen. All subcutaneous injections should be given away from scars, lesions, or moles.

During the patient's acute post op period following repair of an aneurysm, the nurse should ensure that a) hypothermia is maintained to decrease oxygen need. b) the BP and all peripheral pulses are evaluated at least every hour. c) IV fluids are administered at a rate to maintain hourly urine output of 100 mL. d) the patient's BP is kept lower than baseline to prevent leaking at the suture line.

B) the BP and all peripheral pulses are evaluated at least every hour

The client is prescribed digoxin (Lanoxin) for treatment of HR. Which of the following statements by the client indicates the need for further teaching by the nurse? A. "I should not get short of breath anymore." B "This drug will help my heart muscle pump less." C. "I may notice my heart rate decrease." D. "I may feel tired during early treatment."

B. "This drug will help my heart muscle pump less." Rationale: The ability to increase the strength of contractions is a characteristic of cardiac glycosides. It may result in a decrease in pulse. Initially the client may experience some fatigue. Symptoms of CHF, such as dyspnea, should improve.

The healthcare provider is performing an assessment on a patient who is taking propranolol (Inderal) for supraventricular tachycardia. Which assessment finding is an indication the patient is experiencing an adverse effect of this drug? A. Dry mouth B. Bradycardia C. Urinary retention D. Paresthesia

B. Bradycardia

Lisinopril (Prinivil) is part of the treatment regimen for a client with HF. The nurse monitors the client for which electrolyte imbalance of this drug? A. Hyponatremia B. Hyperkalemia C. Hypokalemia D. Hypernatremia

B. Hyperkalemia Rationale: ACE inhibitors block aldosterone secretion, which results in sodium loss and potassium retention. Hyperkalemia may occur, especially when the drug is taken concurrently with potassium-sparing diuretics.

When caring for a patient with a cardiac dysrhythmia, which laboratory value is a priority for the healthcare provider to monitor? A. BUN and creatinine B. Sodium, potassium, and calcium C. Hemoglobin and hematocrit D. PT and INR

B. Sodium, potassium, and chloride

A patient previously lived in a rural part of Hungary where cooking with animal fat was common. Now in her 80s, she lives with her daughter's family in the United States. She has just learned that she has coronary artery disease. To best meet her physical needs, the nurse should A. suggest that she adopt a low-fat vegetarian dietary pattern. B. ask how she usually prepares foods and suggest ways to reduce the use of animal fat. C. advise her to avoid adding fat to foods and eat only skinless breast of chicken and fish as animal foods. D. advise her to eat more fruits and vegetables and to replace some of the animal fat with palm and coconut oil.

B. ask how she usually prepares foods and suggest ways to reduce the use of animal fat.

A resting heart rate of 55 beats/min is a normal finding in a patient who A. is obese. B. is an athlete. C. is taking a diuretic. D. weighs less than 90 lb.

B. is an athlete.

Compared with pain from an MI, angina pain usually A. is unrelieved by rest. B. lasts less than 15 minutes. C. is accompanied by palpitations. D. radiates to the arm, jaw, or throat.

B. lasts less than 15 minutes.

A pt is taking digoxin and furosemide (Lasix) to manage congestive heart failure. The nurse determiens that the pt understands diet therapy when the pt makes which meal choice? A. veggie beef soup, mac and cheese, and a roll B. beef ravioli w/ bread C. baked white fish, mashed potatoes, and carrot salad D. roasted chicken, brown rice, and stewed tomatoes

C

A student nurse is asked to give an example of a long-acting nitrate. She is correct by saying: A. Nitroglycerin sublingual B. nitroglycerin IV C. isosorbide PO D. Nitroglycerin transmucosal

C

A cardiac/vascular nurse reviews recommended activities with a patient who sustained a myocardial infarction. The patient states, "It doesn't really matter what I do or don't do. I will either get better or die." This statement reflects: A. acceptance of changed health status. B. an internal locus of control. C. feelings of loss of control. D. projection.

C

A healthcare provider orders NTG to be administered by IV drip. The nurse carrying out the order would monitor for which of the following as a priority specific to medication? A. shortness of breath when raising head of bed B. urine output C. blood pressure and heart rate D. facial flushing and headache

C

A nurse teaching a pt on correct self administration of sublingual nitroglycerin (Nitrostat) tablets realizes the pt requires further teaching when they state: A. I will call an ambulance while administering the 3rd tablet B. I will not drive while taking nitroglycerin tablets C. I will keep my tablets in a clear bottle D. I will not take nitroglycerin with erectile dysfunction medication

C

A nurse with adequate knowledge about the administration of IV nitroglycerin will recognize that which of the following statements is correct? A. the intravenous form is given by bolus injection B. because the IV forms are short-lived, the dosing must be every 2 hours C. IV nitroglycerin must be protected form exposure to light through use of special tubing D. IV nitroglycerin can be given via gravity drip infusions

C

A patient with a history of coronary artery disease is being treated for a myocardial infarction (MI). During treatment, acute mitral valve regurgitation occurs. What is the most likely cause of the acute mitral valve dysfunction? Please choose from one of the following options. A. Ventricular fibrillation B. Infective endocarditis C. Rupture of the chordae tendinae D. Atherosclerosis

C

The action of medication is inotropic when it: A. decreases afterload B. increases heart rate C. increases the force of contraction D. is uses to treat CHF

C

A patient with chronic heart failure who has been following a low-sodium diet tells the nurse at the clinic about a 5-pound weight gain in the last 3 days. The nurse's first action will be to a. ask the patient to recall the dietary intake for the last 3 days because there may be hidden sources of sodium in the patient's diet. b. instruct the patient in a low-calorie, low-fat diet because the weight gain has likely been caused by excessive intake of inappropriate foods. c. assess the patient for clinical manifestations of acute heart failure because an exacerbation of the chronic heart failure may be occurring. d. educate the patient about the use of diuretic therapy because it is likely that the patient will need medications to reduce the hypervolemia.

C Rationale: The 5-pound weight gain over 3 days indicates that the patient's chronic heart failure may be worsening; it is important that the patient be immediately assessed for other clinical manifestations of decompensation, such as lung crackles. A dietary recall to detect hidden sodium in the diet and teaching about diuretic therapy are appropriate interventions but are not the first nursing actions indicated. There is no evidence that the patient's weight gain is caused by excessive dietary intake of fat or calories, so the answer beginning "instruct the patient in a low-calorie, low-fat diet" describes an inappropriate action

A client with no history of cardiovascular disease comes into the ambulatory clinic with flu-like symptoms. The client suddenly complains of chest pain. Which of the following questions would best help a nurse to discriminate pain caused by a non-cardiac problem? A. "Have you ever had this pain before?" B. "Can you describe the pain to me?" C. "Does the pain get worse when you breathe in?" D. "Can you rate the pain on a scale of 1-10, with 10 being the worst?"

C Chest pain is assessed by using the standard pain assessment parameters. Options 1, 2, and 4 may or may not help discriminate the origin of pain. Pain of pleuropulmonary origin usually worsens on inspiration.

A client enters the ER complaining of severe chest pain. A myocardial infarction is suspected. A 12 lead ECG appears normal, but the doctor admits the client for further testing until cardiac enzyme studies are returned. All of the following will be included in the nursing care plan. Which activity has the highest priority? A. Monitoring vital signs B. Completing a physical assessment C. Maintaining cardiac monitoring D. Maintaining at least one IV access site

C Even though initial tests seem to be within normal range, it takes at least 3 hours for the cardiac enzyme studies to register. In the meantime, the client needs to be watched for bradycardia, heart block, ventricular irritability, and other arrhythmias. Other activities can be accomplished around the MI monitoring.

One hour after administering IV furosemide (Lasix) to a client with heart failure, a short burst of ventricular tachycardia appears on the cardiac monitor. Which of the following electrolyte imbalances should the nurse suspect? A. Hypocalcemia B. Hypermagnesemia C. Hypokalemia D. Hypernatremia

C Furosemide is a potassium-depleting diuretic than can cause hypokalemia. In turn, hypokalemia increases myocardial excitability, leading to ventricular tachycardia.

During the previous few months, a 56-year-old woman felt brief twinges of chest pain while working in her garden and has had frequent episodes of indigestion. She comes to the hospital after experiencing severe anterior chest pain while raking leaves. Her evaluation confirms a diagnosis of stable angina pectoris. After stabilization and treatment, the client is discharged from the hospital. At her follow-up appointment, she is discouraged because she is experiencing pain with increasing frequency. She states that she is visiting an invalid friend twice a week and now cannot walk up the second flight of steps to the friend's apartment without pain. Which of the following measures that the nurse could suggest would most likely help the client deal with this problem? A. Visit her friend earlier in the day B. Rest for at least an hour before climbing the stairs C. Take a nitroglycerin tablet before climbing the stairs D. Lie down once she reaches the friend's apartment

C Nitroglycerin may be used prophylactically before stressful physical activities such as stair climbing to help the client remain pain free. Visiting her friend early in the day would have no impact on decreasing pain episodes. Resting before or after an activity is not as likely to help prevent an activity-related pain episode.

To evaluate a client's condition following cardiac catheterization, the nurse will palpate the pulse: A. In all extremities B. At the insertion site C. Distal to the catheter insertion D. Above the catheter insertion

C Palpating pulses distal to the insertion site is important to evaluate for thrombophlebitis and vessel occlusion. They should be bilateral and strong.

A 57-year-old client with a history of asthma is prescribed propranolol (Inderal) to control hypertension. Before administered propranolol, which of the following actions should the nurse take first? A. Monitor the apical pulse rate B. Instruct the client to take medication with food C. Question the physician about the order D. Caution the client to rise slowly when standing

C Propranolol and other beta-adrenergic blockers are contraindicated in a client with asthma, so the nurse should question the physician before giving the dose. The other responses are appropriate actions for a client receiving propranolol, but questioning the physician takes priority. The client's apical pulse should always be checked before giving propranolol; if the pulse rate is extremely low, the nurse should withhold the drug and notify the physician.

A client is experiencing tachycardia. The nurse's understanding of the physiological basis for this symptom is explained by which of the following statements? A. The demand for oxygen is decreased because of pleural involvement B. The inflammatory process causes the body to demand more oxygen to meet its needs C. The heart has to pump faster to meet the demand for oxygen when there is lowered arterial oxygen tension D. Respirations are labored

C The arterial oxygen supply is lowered and the demand for oxygen is increased, which results in the heart's having to beat faster to meet the body's needs for oxygen.

Which of the following factors can cause blood pressure to drop to normal levels? A. Kidneys' excretion of sodium only B. Kidneys' retention of sodium and water C. Kidneys' excretion of sodium and water D. Kidneys' retention of sodium and excretion of water

C The kidneys respond to a rise in blood pressure by excreting sodium and excess water. This response ultimately affects systolic pressure by regulating blood volume.

A client is at risk for pulmonary embolism and is on anticoagulant therapy with warfarin (Coumadin). The client's prothrombin time is 20 seconds, with a control of 11 seconds. The nurse assesses that this result is: A. The same as the client's own baseline level B. Lower than the needed therapeutic level C. Within the therapeutic range D. Higher than the therapeutic range

C The therapeutic range for prothrombin time is 1.5 to 2 times the control for clients at risk for thrombus. Based on the client's control value, the therapeutic range for this individual would be 16.5 to 22 seconds. Therefore the result is within therapeutic range.

What is the most common complication of a myocardial infarction? A. Cardiogenic shock B. Heart failure C. Arrhythmias D. Pericarditis

C Arrhythmias, caused by oxygen deprivation to the myocardium, are the most common complication of an MI. cardiogenic shock, another complication of MI, is defined as the end stage of left ventricular dysfunction. The condition occurs in approximately 15% of clients with MI. Because the pumping function of the heart is compromised by an MI, heart failure is the second most common complication. Pericarditis most commonly results from a bacterial of viral infection but may occur after MI.

Which of the following complications of an abdominal aortic repair is indicated by detection of a hematoma in the perineal area? A. Hernia B. Stage 1 pressure ulcer C. Retroperitoneal rupture at the repair site D. Rapid expansion of the aneurysm

C Blood collects in the retroperitoneal space and is exhibited as a hematoma in the perineal area. This rupture is most commonly caused by leakage at the repair site. A hernia doesn't cause vascular disturbances, nor does a pressure ulcer. Because no bleeding occurs with rapid expansion of the aneurysm, a hematoma won't form.

Which of the followng conditions is most commonly responsible for myocardial infarction? A. Aneurysm B. Heart failure C. Coronary artery thrombosis D. Renal failure

C Coronary artery thrombosis causes occlusion of the artery, leading to myocardial death. An aneurysm is an outpouching of a vessel and doesn't cause an MI. Renal failure can be associated with MI but isn't a direct cause. Heart failure is usually the result of an MI.

A nurse is viewing the cardiac monitor in a client's room and notes that the client has just gone into ventricular tachycardia. The client is awake and alert and has good skin color. The nurse would prepare to do which of the following? A. Immediately defibrillate B. Prepare for pacemaker insertion C. Administer amiodarone (Cordarone) intravenously D. Administer epinephrine (Adrenaline) intravenously

C First-line treatment of ventricular tachycardia in a client who is hemodynamically stable is the use of anti-dysrhythmics such as amiodarone (Cordarone), lidocaine (Xylocaine), and procainamide (Pronestyl). Cardioversion also may be needed to correct the rhythm (cardioversion is recommended for stable ventricular tachycardia). Defibrillation is used with pulseless ventricular tachycardia. Epinephrine would stimulate and already excitable ventricle and is contraindicated.

When auscultating the apical pulse of a client who has atrial fibrillation, the nurse would expect to hear a rhythm that is characterized by: A. The presence of occasional coupled beats B. Long pauses in an otherwise regular rhythm C. A continuous and totally unpredictable irregularity D. Slow but strong and regular beat

C In atrial fibrillation, multiple ectopic foci stimulate the atria to contract. The AV node is unable to transmit all of these impulses to the ventricles, resulting in a pattern of highly irregular ventricular contractions.

Septal involvement occurs in which type of cardiomyopathy? A. Congestive B. Dilated C. Hypertrophic D. Restrictive

C In hypertrophic cardiomyopathy, hypertrophy of the ventricular septum - not the ventricle chambers - is apparent. This abnormality isn't seen in other types of cardiomyopathy.

Which of the following symptoms might a client with right-sided heart failure exhibit? A. Adequate urine output B. Polyuria C. Oliguria D. Polydipsia

C Inadequate deactivation of aldosterone by the liver after right-sided heart failure leads to fluid retention, which causes oliguria. Adequate urine output, polyuria, and polydipsia aren't associated with right-sided heart failure.

What position should the nurse place the head of the bed in to obtain the most accurate reading of jugular vein distention? A. High-fowler's B. Raised 10 degrees C. Raised 30 degrees D. Supine position

C Jugular venous pressure is measured with a centimeter ruler to obtain the vertical distance between the sternal angle and the point of highest pulsation with the head of the bed inclined between 15 and 30 degrees. Inclined pressure can't be seen when the client is supine or when the head of the bed is raised 10 degrees because the point that marks the pressure level is above the jaw (therefore, not visible). In high Fowler's position, the veins would be barely discernible above the clavicle.

Which hereditary disease is most closely linked to aneurysm? A. Cystic fibrosis B. Lupus erythematosus C. Marfan's syndrome D. Myocardial infarction

C Marfan's syndrome results in the degeneration of the elastic fibers of the aortic media. Therefore, clients with the syndrome are more likely to develop an aortic aneurysm. Although cystic fibrosis is hereditary, it hasn't been linked to aneurysms. Lupus erythematosus isn't hereditary. Myocardial infarction is neither hereditary nor a disease.

A 68-year-old woman is scheduled to undergo mitral valve replacement for severe mitral stenosis and mitral regurgitation. Although the diagnosis was made during childhood, she did not have any symptoms until 4 years ago. Recently, she noticed increased symptoms, despite daily doses of digoxin and furosemide. During the initial interview with the nice lady, the nurse would most likely learn that the client's childhood health history included: A. Chicken pox B. poliomyelitis C. Rheumatic fever D. meningitis

C Most clients with mitral stenosis have a history of rheumatic fever or bacterial endocarditis.

An elderly patient with a 40-pack-year history of smoking and a recent myocardial infarction is admitted to the medical unit with acute shortness of breath; the nurse need to rule out pneumonia versus heart failure. The diagnostic test that the nurse will monitor to help in determining whether the patient has heart failure is a. 12-lead electrocardiogram (ECG). b. arterial blood gases (ABGs). c. B-type natriuretic peptide (BNP). d. serum creatine kinase (CK).

C Rationale: BNP is secreted when ventricular pressures increase, as with heart failure, and elevated BNP indicates a probable or very probable diagnosis of heart failure. 12-lead ECGs, ABGs, and CK may also be used in determining the causes or effects of heart failure but are not as clearly diagnostic of heart failure as BNP.

A home health care patient has recently started taking oral digoxin (Lanoxin) and furosemide (Lasix) for control of heart failure. The patient data that will require the most immediate action by the nurse is if the patient's a. weight increases from 120 pounds to 122 pounds over 3 days. b. liver is palpable 2 cm below the ribs on the right side. c. serum potassium level is 3.0 mEq/L after 1 week of therapy. d. has 1 to 2+ edema in the feet and ankles in the morning.

C Rationale: Hypokalemia potentiates the actions of digoxin and increases the risk for digoxin toxicity, which can cause life-threatening dysrhythmias. The other data indicate that the patient's heart failure requires more effective therapies, but they do not require nursing action as rapidly as the low serum potassium level.

During assessment of a 72-year-old with ankle swelling, the nurse notes jugular venous distention (JVD) with the head of the patient's bed elevated 45 degrees. The nurse knows this finding indicates a. decreased fluid volume. b. incompetent jugular vein valves. c. elevated right atrial pressure. d. jugular vein atherosclerosis.

C Rationale: The jugular veins empty into the superior vena cava and then into the right atrium, so JVD with the patient sitting at a 45-degree angle reflects elevated right atrial pressure. JVD is an indicator of excessive fluid volume (increased preload), not decreased fluid volume; it is not caused by incompetent jugular vein valves or atherosclerosis.

During a visit to an elderly patient with chronic heart failure, the home care nurse finds that the patient has severe dependent edema and that the legs appear to be weeping serous fluid. Based on these data, the best nursing diagnosis for the patient is a. activity intolerance related to venous congestion. b. disturbed body image related to massive leg swelling. c. impaired skin integrity related to peripheral edema. d. impaired gas exchange related to chronic heart failure.

C Rationale: The patient's findings of severe dependent edema and weeping serous fluid from the legs support the nursing diagnosis of impaired skin integrity. There is less evidence for the nursing diagnoses of activity intolerance, disturbed body image, and impaired gas exchange, although the nurse will further assess the patient to determine whether there are other clinical manifestations of heart failure to indicate that these diagnoses are appropriate.

What supplemental medication is most frequently ordered in conjuction with furosemide (Lasix)? A. Chloride B. Digoxin C. Potassium D. Sodium

C Supplemental potassium is given with furosemide because of the potassium loss that occurs as a result of this diuretic. Chloride and sodium aren't loss during diuresis. Digoxin acts to increase contractility but isn't given routinely with furosemide.

When assessing a client for an abdominal aortic aneurysm, which area of the abdomen is most commonly palpated? A. Right upper quadrant B. Directly over the umbilicus C. Middle lower abdomen to the left of the midline D. Midline lower abdomen to the right of the midline

C The aorta lies directly left of the umbilicus; therefore, any other region is inappropriate for palpation.

Which of the following blood vessel layers may be damaged in a client with an aneurysm? A. Externa B. Interna C. Media D. Interna and Media

C The factor common to all types of aneurysms is a damaged media. The media has more smooth muscle and less elastic fibers, so it's more capable of vasoconstriction and vasodilation. The interna and externa are generally no damaged in an aneurysm.

Which of the following arteries primarily feeds the anterior wall of the heart? A. Circumflex artery B. Internal mammary artery C. Left anterior descending artery D. Right coronary artery

C The left anterior descending artery is the primary source of blood for the anterior wall of the heart. The circumflex artery supplies the lateral wall, the internal mammary artery supplies the mammary, and the right coronary artery supplies the inferior wall of the heart.

Prolonged occlusion of the right coronary artery produces an infarction in which of he following areas of the heart? A. Anterior B. Apical C. Inferior D. Lateral

C The right coronary artery supplies the right ventricle, or the inferior portion of the heart. Therefore, prolonged occlusion could produce an infarction in that area. The right coronary artery doesn't supply the anterior portion ( left ventricle ), lateral portion ( some of the left ventricle and the left atrium ), or the apical portion ( left ventricle ) of the heart.

Which of the following blood tests is most indicative of cardiac damage? A. Lactate dehydrogenase B. Complete blood count C. Troponin I D. Creatine kinase

C Troponin I levels rise rapidly and are detectable within 1 hour of myocardial injury. Troponin I levels aren't detectable in people without cardiac injury. Lactate dehydrogenase is present in almost all body tissues and not specific to heart muscle. LDH isoenzymes are useful in diagnosing cardiac injury. CBC is obtained to review blood counts, and a complete chemistry is obtained to review electrolytes. Because CK levels may rise with skeletal muscle injury, CK isoenzymes are required to detect cardiac injury.

When teaching a client with a cardiac problem, who is on a high-unsaturated fatty-acid diet, the nurse should stress the importance of increasing the intake of: A. Enriched whole milk B. Red meats, such as beef C. Vegetables and whole grains D. Liver and other glandular organ meats

C Vegetables and whole grains are low in fat and may reduce the risk for heart disease.

A surgical repair is planned for a patient who has a 5-cm abdominal aortic aneurysm (AAA). On physical assessment of the patient, the nurse would expect to find... A) hoarseness and dysphagia B) sever back pain with flank ecchymosis C) the presence of a bruit in the periumbilical area D) weakness in the lower extremities progressing to paraplegia

C) the presence of a bruit in the periumbilical area, although most abdominal aortic aneurysms (AAA) are asymptomatic, on physical exam a pulsatile mass in the periumbilical area slightly to the left of the midline may be detected, and bruits may be audible with a stethoscope placed over the aneurysm. Hoarseness and dysphagia may occur with aneurysms of the ascending aorta and the aortic arch. Severe back pain with flank ecchymosis is usually present on rupture of an AAA, and neurologic loss in the lower extremities may occur from the pressure of a thoracic aneurysm.

If a nurse is teaching foot care to a patient with peripheral vascular disease, which of the following is a correctly written learning objective? A. "The nurse will instruct the patient on appropriate foot care." B. "The nurse will demonstrate to the patient the proper technique for trimming toenails." C. "By discharge, the patient will list three ways to protect the feet from injury." D. "The patient will understand the rationale for proper foot care after instruction

C. "By discharge, the patient will list three ways to protect the feet from injury."

A 38-year-old man is treated for hypertension with amiloride/hydrochlorothiazide (Maxzide) and metoprolol (Lopressor). Four months after his last clinic visit, his BP returns to pretreatment levels and he admits he has not been taking his medication regularly. The best response by the nurse is, A. "Try always to take your medication when you carry out another daily routine so you do not forget to take it." B. "If you would exercise more and stop smoking, you probably would not need to be taking medications for hypertension." C. "The drugs you are taking cause sexual dysfunction in many patients. Are you experiencing any problems in this area?" D. "You need to remember that hypertension can be only controlled with medication, not cured, and you must always take your medication."

C. "The drugs you are taking cause sexual dysfunction in many patients. Are you experiencing any problems in this area?"

Which of the following statements would indicate that a patient taking antihypertensive drugs understands the management of hypertension? A. "I need to have my blood pressure checked monthly." B. "I can still smoke while taking these drugs, as long as I cut down." C. "These pills will help control, but not cure, my high blood pressure." D. "When my blood pressure is back to normal, I can stop taking these pills."

C. "These pills will help control, but not cure, my high blood pressure."

Distant, muffled heart sounds and distended neck veins may indicate A. flail chest. B. ruptured spleen. C. cardiac tamponade. D. thoracic vertebral fracture.

C. cardiac tamponade.

The client's serum digoxin level is 2.2 ng/dL and the heart rate is 120 and irregular. The nurse expects to administer which of the following drugs? A. Digoxin immune Fab (Digibind) B. Furosemide (Lasix) 60 mg I.V. C. Digoxin 0.5 mg bolus I.V. D. Potassium 40 mEq added to I.V. fluids

C. Digoxin immune Fab (Digibind) Rationale: Digibind binds and removes digoxin from the body and prevents toxic effects of digoxin overdose. A serum level of 2.2 is elevated, and the client is exhibiting signs of digoxin toxicity. The question does not indicate that the potassium level is low. Giving additional digoxin would exacerbate the toxicity. Giving Lasix may reduce potassium levels and contribute to increased toxicity.

The nurse developing a teaching plan for a client receiving thiazide diuretics should include the following. A. Teaching the client to take apical pulse. B. Decreasing potassium-rich foods in the diet. C. Including citrus fruits, melons, and vegetables in the diet. D. Teaching the client to check blood pressure t.i.d.

C. Including citrus fruits, melons, and vegetables in the diet. Rationale: Thiazide diuretics are potassium wasting, and levels should be closely monitored. Encouraging foods rich in potassium could help maintain potassium levels. Taking an apical pulse is indicated before administering cardiac glycosides and beta blockers. It would not be necessary to check blood pressure TID unless the client was experiencing hypotension.

The nurse reviews lab studies of a client receiving digoxin (Lanoxin). Intervention by the nurse is required if the results include a A.Serum sodium level of 140 mEq/L. B. Serum digoxin level of 1.2 ng/dL. C.Serum potassium level of 3.0 mEq/L. D. Hemoglobin 14.4 g/dL.

C. Serum potassium level of 3.0 mEq/L. Rationale: Normal serum potassium level is 3.5-5.0 mEq/L. Hypokalemia may predispose the client to digitalis toxicity. The other lab values are WNL.

The client is prescribed a beta-blocker as adjunct therapy to treatment of heart failure. The nurse recognizes that beta blockers act by A. Increasing contractility and cardiac output. B. Decreasing preload. C. Slowing the heart and decreasing afterload. D. Decreasing peripheral resistance

C. Slowing the heart and decreasing afterload. Rationale: Beta-blockers improve symptoms of HF by slowing heart rate and decreasing blood pressure. The decreased afterload causes decreased workload on the heart.

When a patient reports chest pain, unstable angina must be identified and treated because A. the pain may be severe and disabling. B. ECG changes and dysrhythmias may occur during an attack. C. atherosclerotic plaque deterioration may cause complete thrombus of the vessel lumen. D. the spasm of a major coronary artery may cause total occlusion of the vessel with progression to MI.

C. atherosclerotic plaque deterioration may cause complete thrombus of the vessel lumen.

A patient is scheduled for exercise nuclear imaging stress testing. The nurse explains to the patient that this test involves A. placement of electrodes inside the right-sided heart chambers through a vein to record the electrical activity of the heart directly. B. exercising on a treadmill or stationary bicycle with continuous ECG (electrocardiographic) monitoring to detect ischemic changes during exercise. C. intravenous (IV) administration of a radioisotope of technetium-99 sestamibi at the maximum HR during exercise to identify areas of cardiac damage. D. placement of a small transducer in four positions on the chest to record the direction and flow of blood through the heart by the reflection of sound waves.

C. intravenous (IV) administration of a radioisotope of technetium-99 sestamibi at the maximum HR during exercise to identify areas of cardiac damage.

A patient is experiencing a heart rate of 200 beats/min. If the ECG pattern demonstrates absent P waves and normal and consistent QRS complexes and duration, a nurse should interpret these findings as indicative of A. ventricular tachycardia. B. second-degree heart block. C. supraventricular tachycardia. D. premature ventricular contractions

C. supraventricular tachycardia.

The patient has CVI and a venous ulcer. The unlicensed assistive personnel (UAP) decides to apply compression stockings because that is what these patients always have ordered. What assessment by the nurse would cause the application of compression stockings to harm the patient? a Rest pain b High blood pressure c Elevated blood sugar d Dry, itchy, flaky skin

Correct Answer: A Rationale: Rest pain occurs as peripheral artery disease (PAD) progresses and involves multiple arterial segments. Compression stockings should not be used on patients with PAD. Elevated blood glucose, possibly indicating uncontrolled diabetes mellitus, and hypertension may or may not indicate arterial problems. Dry, itchy, flaky skin indicates venous insufficiency. The RN should be the one to obtain the order and instruct the UAP to apply compression stockings if they are ordered.

When the patient is being examined for venous thromboembolism (VTE) in the calf, what diagnostic test should the nurse expect to teach the patient about first? a Duplex ultrasound b Contrast venography c Magnetic resonance venography d Computed tomography venography

Correct Answer: A Rationale: The duplex ultrasound is the most widely used test to diagnose VTE. Contrast venography is rarely used now. Magnetic resonance venography is less accurate for calf veins than pelvic and proximal veins. Computed tomography venography may be used but is invasive and much more expensive than the duplex ultrasound.

Which person should the nurse identify as having the highest risk for abdominal aortic aneurysm? a A 70-year-old male, with high cholesterol and hypertension Correct b A 40-year-old female with obesity and metabolic syndrome c A 60-year-old male with renal insufficiency who is physically inactive d A 65-year-old female with hyperhomocysteinemia and substance abuse

Correct Answer: A The most common etiology of descending abdominal aortic aneurysm (AAA) is atherosclerosis. Male gender, age 65 years or older, and tobacco use are the major risk factors for AAAs of atherosclerotic origin. Other risk factors include the presence of coronary or peripheral artery disease, high blood pressure, and high cholesterol.

A 73-year-old man with dementia has a venous ulcer related to chronic venous insufficiency. The nurse should provide education on which type of diet for this patient and his caregiver? a Low-fat diet b High-protein diet c Calorie-restricted diet d High-carbohydrate diet

Correct Answer: B A patient with a venous ulcer should have a balanced diet with adequate protein, calories, and micronutrients; this type of diet is essential for healing. Nutrients most important for healing include protein, vitamins A and C, and zinc. Foods high in protein (e.g., meat, beans, cheese, tofu), vitamin A (green leafy vegetables), vitamin C (citrus fruits, tomatoes, cantaloupe), and zinc (meat, seafood) must be provided. Restricting fat or calories is not helpful for wound healing or in patients of normal weight. For overweight individuals with no active venous ulcer, a weight-loss diet should be considered.

A 67-year-old man with peripheral artery disease is seen in the primary care clinic. Which symptom reported by the patient would indicate to the nurse that the patient is experiencing intermittent claudication? a Patient complains of chest pain with strenuous activity. b Patient says muscle leg pain occurs with continued exercise. c Patient has numbness and tingling of all his toes and both feet. d Patient states the feet become red if he puts them in a dependent position.

Correct Answer: B Intermittent claudication is an ischemic muscle ache or pain that is precipitated by a consistent level of exercise, resolves within 10 minutes or less with rest, and is reproducible. Angina is the term used to describe chest pain with exertion. Paresthesia is the term used to describe numbness or tingling in the toes or feet. Reactive hyperemia is the term used to describe redness of the foot; if the limb is in a dependent position the term is dependent rubor.

A patient was just diagnosed with acute arterial ischemia in the left leg secondary to atrial fibrillation. Which early clinical manifestation must be reported to the physician immediately to save the patient's limb? a Paralysis b Paresthesia c Crampiness d Referred pain

Correct Answer: B Rationale: The physician must be notified immediately if any of the six Ps of acute arterial ischemia occur to prevent ischemia from quickly progressing to tissue necrosis and gangrene. The six Ps are paresthesia, pain, pallor, pulselessness, and poikilothermia, with paralysis being a very late sign indicating the death of nerves to the extremity. Crampy leg sensation is more common with varicose veins. The pain is not referred.

The nurse is caring for a newly admitted patient with vascular insufficiency. The patient has a new order for enoxaparin (Lovenox) 30 mg subcutaneously. What should the nurse do to correctly administer this medication? a. Spread the skin before inserting the needle. b. Leave the air bubble in the prefilled syringe. c. Use the back of the arm as the preferred site. d. Sit the patient at a 30-degree angle before administration.

Correct Answer: B Rationale: The nurse should not expel the air bubble from the prefilled syringe because it should be injected to clear the needle of medication and avoid leaving medication in the needle track in the tissue.

A patient with varicose veins has been prescribed compression stockings. How should the nurse teach the patient to use these? a. "Try to keep your stockings on 24 hours a day, as much as possible." b. "While you're still lying in bed in the morning, put on your stockings." c. "Dangle your feet at your bedside for 5 minutes before putting on your stockings." d. "Your stockings will be most effective if you can remove them for a few minutes several times a day."

Correct Answer: B Rationale: The patient with varicose veins should apply stockings in bed, before rising in the morning. Stockings should not be worn continuously, but they should not be removed several times daily. Dangling at the bedside prior to application is likely to decrease their effectiveness.

The patient had aortic aneurysm repair. What priority nursing action will the nurse use to maintain graft patency? a Assess output for renal dysfunction. b Use IV fluids to maintain adequate BP. c Use oral antihypertensives to maintain cardiac output. d Maintain a low BP to prevent pressure on surgical site

Correct Answer: B Rationale: The priority is to maintain an adequate BP (determined by the surgeon) to maintain graft patency. A prolonged low BP may result in graft thrombosis, and hypertension may cause undue stress on arterial anastomoses resulting in leakage of blood or rupture at the suture lines, which is when IV antihypertensives may be used. Renal output will be assessed when the aneurysm repair is above the renal arteries to assess graft patency, not maintain it.

A female patient with critical limb ischemia has had peripheral artery bypass surgery to improve her circulation. What care should the nurse provide on postoperative day 1? a Keep the patient on bed rest. b Assist the patient with walking several times. c Have the patient sit in the chair several times. d Place the patient on her side with knees flexed

Correct Answer: B Rationale: To avoid blockage of the graft or stent, the patient should walk several times on postoperative day 1 and subsequent days. Having the patient's knees flexed for sitting in a chair or in bed increase the risk of venous thrombosis and may place stress on the suture lines.

The nurse would determine that a postoperative patient is not receiving the beneficial effects of enoxaparin (Lovenox) after noting what during a routine shift assessment? a. Generalized weakness and fatigue b. Crackles bilaterally in the lung bases c. Pain and swelling in lower extremity d. Abdominal pain with decreased bowel sounds

Correct Answer: C Rationale: Enoxaparin is a low-molecular-weight heparin used to prevent the development of deep vein thromboses (DVTs) in the postoperative period. Pain and swelling in the lower extremity can indicate development of DVT and therefore may signal ineffective medication therapy.

The nurse is caring for a preoperative patient who has an order for vitamin K by subcutaneous injection. The nurse should verify that which laboratory study is abnormal before administering the dose? a. Hematocrit (Hct) b. Hemoglobin (Hgb) c. Prothrombin time (PT) d. Partial thromboplastin time (PTT)

Correct Answer: C Rationale: Vitamin K counteracts hypoprothrombinemia and/or reverses the effects of warfarin (Coumadin) and thus decreases the risk of bleeding. High values for either the prothrombin time (PT) or the international normalized ratio (INR) demonstrates the need for this medication.

The nurse is caring for a patient who has been receiving warfarin (Coumadin) and digoxin (Lanoxin) as treatment for atrial fibrillation. Because the warfarin has been discontinued before surgery, the nurse should diligently assess the patient for which complication early in the postoperative period until the medication is resumed? a. Decreased cardiac output b. Increased blood pressure c. Cerebral or pulmonary emboli d. Excessive bleeding from incision or IV sites

Correct Answer: C Rationale: Warfarin is an anticoagulant that is used to prevent thrombi from forming on the walls of the atria during atrial fibrillation. Once the medication is terminated, thrombi could again form. If one or more thrombi detach from the atrial wall, they could travel as cerebral emboli from the left atrium or pulmonary emboli from the right atrium.

A male patient was admitted for a possible ruptured aortic aneurysm, but had no back pain. Ten minutes later his assessment includes the following: sinus tachycardia at 138, BP palpable at 65 mm Hg, increasing waist circumference, and no urine output. How should the nurse interpret this assessment about the patient's aneurysm? a Tamponade will soon occur. b The renal arteries are involved. c Perfusion to the legs is impaired. d He is bleeding into the abdomen.

Correct Answer: D Rationale: The lack of back pain indicates the patient is most likely exsanguinating into the abdominal space, and the bleeding is likely to continue without surgical repair. A blockade of the blood flow will not occur in the abdominal space as it would in the retroperitoneal space where surrounding anatomic structures may control the bleeding. The lack of urine output does not indicate renal artery involvement, but that the bleeding is occurring above the renal arteries, which decreases the blood flow to the kidneys. There is no assessment data indicating decreased perfusion to the legs.

The nurse is preparing to administer a scheduled dose of enoxaparin (Lovenox) 30 mg subcutaneously. What should the nurse do to administer this medication correctly? a. Remove the air bubble in the prefilled syringe. b. Aspirate before injection to prevent IV administration. c. Rub the injection site after administration to enhance absorption. d. Pinch the skin between the thumb and forefinger before inserting the needle

Correct Answer: D Rationale: The nurse should gather together or "bunch up" the skin between the thumb and the forefinger before inserting the needle into the subcutaneous tissue. The nurse should not remove the air bubble in the prefilled syringe, aspirate, nor rub the site after injection.

The nurse is caring for a patient with a recent history of deep vein thrombosis (DVT). The patient now needs to undergo surgery for appendicitis. The nurse is reviewing the laboratory results for this patient before administering an ordered dose of vitamin K. The nurse determines that the medication is both safe to give and is most needed when the international normalized ratio (INR) is which result? a. 1.0 b. 1.2 c. 1.6 d. 2.2

Correct Answer: D Rationale: itamin K is the antidote to warfarin (Coumadin), which the patient has most likely been taking before admission for treatment of DVT. Warfarin is an anticoagulant that impairs the ability of the blood to clot. Therefore it is necessary to give vitamin K before surgery to reduce the risk of hemorrhage. The largest value of the INR indicates the greatest impairment of clotting ability, making 2.2 the correct selection.

A 39-year-old woman with a history of smoking and oral contraceptive use is admitted with a venous thromboembolism (VTE) and prescribed unfractionated heparin. What laboratory test should the nurse review to evaluate the expected effect of the heparin? a Platelet count b Activated clotting time (ACT) c International normalized ratio (INR) d Activated partial thromboplastin time (APTT)

Correct Answer: D Unfractionated heparin can be given by continuous IV for VTE treatment. When given IV, heparin requires frequent laboratory monitoring of clotting status as measured by activated partial thromboplastin time (aPTT). Platelet counts can decrease as an adverse reaction to heparin, but that is not the expected effect.

A nurse is caring for a patient with a diagnosis of deep venous thrombosis (DVT). The patient has an order to receive 30 mg enoxaparin (Lovenox). Which injection site should the nurse use to administer this medication safely? a. Buttock, upper outer quadrant b. Abdomen, anterior-lateral aspect c. Back of the arm, 2 inches away from a mole d. Anterolateral thigh, with no scar tissue nearby

Correct Answer: b Rationale: Enoxaparin (Lovenox) is a low-molecular-weight (LMW) heparin that is given as a deep subcutaneous injection in the right and left anterolateral abdomen. All subcutaneous injections should be given away from scars, lesions, or moles.

What medications should the nurse expect to include in the teaching plan to decrease the risk of cardiovascular events and death for PAD patients (select all that apply)? a Ramipril (Altace) b Cilostazol (Pletal) c Simvastatin (Zocor) d Clopidogrel (Plavix) e Warfarin (Coumadin) f Aspirin (acetylsalicylic acid)

Correct Answers: a, c, d Angiotensin-converting enzyme inhibitors (e.g., ramipril [Altace]) are used to control hypertension. Statins (e.g., simvastatin [Zocor]) are used for lipid management. Aspirin is used as an antiplatelet agent. Cilostazol (Pletal) is used for intermittent claudication, but it does not reduce CVD morbidity and mortality risks. Clopidogrel may be used if the patient cannot tolerate aspirin. Anticoagulants (e.g., warfarin [Coumadin]) are not recommended to prevent CVD events in PAD patients.

A key aspect of teaching for the patient on anticoagulant therapy includes which instructions? a. Monitor for and report any signs of bleeding. b. Do not take acetaminophen (Tylenol) for a headache. c. Decrease your dietary intake of foods containing vitamin K. d. Arrange to have blood drawn routinely to check drug levels.

Correct answer: a Rationale: Patients taking anticoagulants should be taught to monitor and report any signs of bleeding, which can be a serious complication. Other important patient teaching includes maintenance of a consistent intake of foods containing vitamin K, avoidance of supplements that contain vitamin K, and routine coagulation laboratory studies if a patient is taking warfarin.

The patient at highest risk for venous thromboembolism (VTE) is a. a 62-year-old man with spider veins who is having arthroscopic knee surgery. b. a 32-year-old woman who smokes, takes oral contraceptives, and is planning a trip to Europe. c. a 26-year-old woman who is 3 days postpartum and received maintenance IV fluids for 12 hours during her labor. d. an active 72-year-old man at home recovering from transurethral resection of the prostate for benign prostatic hyperplasia.

Correct answer: b Rationale: Three important factors (called Virchow's triad) in the etiology of venous thrombosis are (1) venous stasis, (2) damage of the endothelium (inner lining of the vein), and (3) hypercoagulability of the blood. Patients at risk for venous thrombosis usually have predisposing conditions for these three disorders (see Table 38-8). The 32-year-old woman has the highest risk: long trips without adequate exercise (venous stasis), tobacco use, and use of oral contraceptives. Note: The likelihood of hypercoagulability of blood is increased in women older than 35 years who use tobacco.

Rest pain is a manifestation of PAD that occurs due to a chronic a. vasospasm of small cutaneous arteries in the feet. b. increase in retrograde venous blood flow in the legs. c. decrease in arterial blood flow to the nerves of the feet. d. decrease in arterial blood flow to the leg muscles during exercise.

Correct answer: c Rationale: Rest pain most often occurs in the forefoot or toes and is aggravated by limb elevation. Rest pain occurs when blood flow is insufficient to meet basic metabolic requirements of the distal tissues. Rest pain occurs more often at night because cardiac output tends to drop during sleep and the limbs are at the level of the heart. Patients often try to achieve partial pain relief by dangling the leg over the side of the bed or sleeping in a chair to allow gravity to maximize blood flow.

A patient with infective endocarditis develops sudden left leg pain with pallor, paresthesia, and a loss of peripheral pulses. The nurse's initial action should be to a. elevate the leg to promote venous return. b. start anticoagulant therapy with IV heparin. c. notify the physician of the change in peripheral perfusion. d. place the bed in reverse Trendelenburg to promote perfusion

Correct answer: c Rationale: The patient has potentially developed acute arterial ischemia (sudden interruption in the arterial blood supply to the extremity), caused by an embolism from a cardiac thrombus that occurred as a complication of infective endocarditis. Clinical manifestations of acute arterial ischemia include any or all of the six Ps : pain, pallor, paralysis, pulselessness, paresthesia, and poikilothermia. Without immediate intervention, ischemia may progress quickly to tissue necrosis and gangrene within a few hours. If the nurse detects these signs, the physician should be notified immediately.

Priority nursing measures after an abdominal aortic aneurysm repair include a. assessment of cranial nerves and mental status. b. administration of IV heparin and monitoring of aPTT. c. administration of IV fluids and monitoring of kidney function. d. elevation of the legs and application of elastic compression stockings

Correct answer: c Rationale: Postoperative priorities include administration of IV fluids and maintenance of renal perfusion. An adequate blood pressure is important for maintaining graft patency, and administration of IV fluids and blood components (as indicated) is essential for adequate blood flow. The nurse should evaluate renal function by measuring hourly urine output and monitoring daily blood urea nitrogen (BUN) and serum creatinine levels. Irreversible renal failure may occur after aortic surgery, particularly in individuals at high risk.

The recommended treatment for an initial VTE in an otherwise healthy person with no significant co-morbidities would include a. IV argatroban (Acova) as an inpatient. b. IV unfractionated heparin as an inpatient. c. subcutaneous unfractionated heparin as an outpatient. d. subcutaneous low-molecular-weight heparin as an outpatient.

Correct answer: d Rationale: Patients with confirmed VTE should receive initial treatment with low-molecular-weight heparin (LMWH), unfractionated heparin (UH), fondaparinux, or rivaroxaban, followed by warfarin for 3 months to maintain the international normalized ratio (INR) between 2.0 and 3.0 for 24 hours. Patients with multiple comorbid conditions, complex medical issues, or a very large VTE usually are hospitalized for treatment and typically receive intravenous UH. LMWH only for 3 months is another option for patients with acute VTE. Depending on the clinical presentation, patients often can be managed safely and effectively as outpatients.

Which are probable clinical findings in a person with an acute lower extremity VTE (select all that apply)? a. Pallor and coolness of foot and calf b. Mild to moderate calf pain and tenderness c. Grossly diminished or absent pedal pulses d. Unilateral edema and induration of the thigh e. Palpable cord along a superficial varicose vein

Correct answers: b, d Rationale: The patient with lower extremity venous thromboembolism (VTE) may or may not have unilateral leg edema, extremity pain, a sense of fullness in the thigh or calf, paresthesias, warm skin, erythema, or a systemic temperature greater than 100.4 F (38 C). If the calf is involved, it may be tender to palpation.

A nurse is caring for a pt receiving heparin IV. The nurse anticipates that which lab study will be prescribed to monitor the therapeutic effect of heparin? A. hematocrit B. hemoglobin C. prothrombin time D. activated partial thromboplastin time

D

A nurse provides discharge instructions to a post op pt who is taking warfarin sodium (Coumadin). Which statement, if made by the pt, reflects the need for further teaching? A. I will take my pills every day at the same time B. I will be certain to limit my alcohol consumption C. I have already called my family to pick up my Medic-Alert bracelet D. I will take enteric-coated Aspirin for my headaches

D

During the cardiac assessment, the nurse finds a client has jugular vein distention. What does this mean to the nurse? a. The client is fine. b. The client is dehydrated. c. The client has an infection. d. The client could have fluid overload

D

During the cardiac assessment, the nurse learns a client had rheumatic heart fever as a child. For which of the following cardiac conditions should this client be assessed? a. Pulmonic stenosis b. Aortic stenosis c. Mitral regurgitation d. Mitral stenosis

D

During the examination of an eight-month-pregnant client, the nurse measures the blood pressure at 160/98 and notes bilateral edema of the ankles. What do these findings suggest to the nurse? a. This client is going into labor early. b. The client is in normal health. c. The client is hyperventilating. d. The client could be preeclamptic.

D

Nurse Margie just administered an ACE inhibitor to her pt. Before ambulating the pt for the first time after administration, the nurse should monitor for: A. hypokalemia B. irregular heartbeat C. edema D. hypotension

D

The action of an ACE inhibitor interrupts the renin-angiotensin-aldosterone mechanism, thereby producing which of the following? A. reduced renal blood flow B. reduced sodium and water retention C. increased peripheral vascular resistance D. increased sodium excretion and potassium reabsorption

D

The healthcare provider is examining the electrocardiogram (EKG) of a patient and notes the PR interval is 6 small boxes in length. What is the significance of this finding? A. Stress is causing increased sympathetic stimulation. B. This should be documented as an expected finding. C. There may be some scar tissue in one of the ventricles. D. There may be a delay in the conduction through the AV node.

D

The nurse is scheduled to administer a dose of digoxin to and adult pt with atrial fibrillation. The pt has a potassium level of 4.3 mEq/L. The nurse should perform which of the following activities next? A. withhold dose only for that day B. obtain order for dose of potassium before giving digoxin C. withhold dose and notify prescriber D. administer dose as ordered

D

The therapeutic drug level for digoxin is: A. 0.1-2.0 B. 1.0-2.0 C. 0.1-0.5 D. 0.5-2.0

D

Which of the following is a potential side effect of IV furosemide (Lasix)? A. drowsiness B. diarrhea C. cystitis D. hearing loss

D

Which of the following is a potential side effect of IV furosemide (lasix)? A. drowsiness B. diarrhea C. cystitis D. hearing loss

D

Which of the following pts is at greatest risk for digital toxicity? A. a 25 y/o pt with congestive heart disease B. a 50 y/o pt with CHF C. a 60 y/o pt who had an MI D. an 80 y/o pt with CHF

D

Which of the following pts is most likely to experience adverse effects from tx with diuretics? A. a 21 y/o college student B. a 40 y/o unmarried man C. a 60 y/o widower D. a 75 y/o man

D

While assessing a pt with angina who is to start Beta Blocker therapy, the nurse is aware that the presence of which condition may be a problem if these drugs are used: A. hypertension B. essential tremors C. exertional angina D. asthma

D

Which of the following terms is used to describe the amount of stretch on the myocardium at the end of diastole? A. Afterload B. Cardiac index C. Cardiac output D. Preload

D Preload is the amount of stretch of the cardiac muscle fibers at the end of diastole. The volume of blood in the ventricle at the end of diastole determines the preload. Afterload is the force against which the ventricle must expel blood. Cardiac index is the individualized measurement of cardiac output, based on the client's body surface area. Cardiac output is the amount of blood the heart is expelling per minute.

When assessing an ECG, the nurse knows that the P-R interval represents the time it takes for the: A. Impulse to begin atrial contraction B. Impulse to transverse the atria to the AV node C. SA node to discharge the impulse to begin atrial depolarization D. Impulse to travel to the ventricles

D The P-R interval is measured on the ECG strip from the beginning of the P wave to the beginning of the QRS complex. It is the time it takes for the impulse to travel to the ventricle.

IV heparin therapy is ordered for a client. While implementing this order, a nurse ensures that which of the following medications is available on the nursing unit? A. Vitamin K B. Aminocaproic acid C. Potassium chloride D. Protamine sulfate

D The antidote to heparin is protamine sulfate and should be readily available for use if excessive bleeding or hemorrhage should occur. Vitamin K is an antidote for warfarin.

Which of the following types of pain is most characteristic of angina? A. Knifelike B. Sharp C. Shooting D. Tightness

D The pain of angina usually ranges from a vague feeling of tightness to heavy, intense pain. Pain impulses originate in the most visceral muscles and may move to such areas as the chest, neck, and arms.

Which of the following cardiac conditions does a fourth heart sound (S4) indicate? A. Dilated aorta B. Normally functioning heart C. Decreased myocardial contractility D. Failure of the ventricle to eject all the blood during systol

D An S4 occurs as a result of increased resistance to ventricular filling after atrial contraction. This increased resistance is related to decrease compliance of the ventricle. A dilated aorta doesn't cause an extra heart sound, though it does cause a murmur. Decreased myocardial contractility is heard as a third heart sound. An s4 isn't heard in a normally functioning heart.

After myocardial infarction, serum glucose levels and free fatty acids are both increase. What type of physiologic changes are these? A. Electrophysiologic B. Hematologic C. Mechanical D. Metabolic

D Both glucose and fatty acids are metabolites whose levels increase after a myocardial infarction. Mechanical changes are those that affect the pumping action of the heart, and electro physiologic changes affect conduction. Hematologic changes would affect the blood.

A client with a bundle branch block is on a cardiac monitor. The nurse should expect to observe: A. Sagging ST segments B. Absence of P wave configurations C. Inverted T waves following each QRS complex D. Widening of QRS complexes to 0.12 second or greater

D Bundle branch block interferes with the conduction of impulses from the AV node to the ventricle supplied by the affected bundle. Conduction through the ventricles is delayed, as evidenced by a widened QRS complex.

Cholesterol, frequently discussed in relation to atherosclerosis, is a substance that: A. May be controlled by eliminating food sources B. Is found in many foods, both plant and animal sources C. All persons would be better off without because it causes the disease process D. Circulates in the blood, the level of which usually decreases when unsaturated fats are substituted for saturated fats

D Cholesterol is a sterol found in tissue; it is attributed in part to diets high in saturated fats.

Exceeding which of the following serum cholesterol levels significantly increases the risk of coronary artery disease? A. 100 mg/dl B. 150 mg/dl C. 175 mg/dl D. 200 mg/dl

D Cholesterol levels above 200 mg/dl are considered excessive. They require dietary restriction and perhaps medication. Exercise also helps reduce cholesterol levels. The other levels listed are all below the nationally accepted levels for cholesterol and carry a lesser risk for CAD.

The adaptations of a client with complete heart block would most likely include: A. Nausea and vertigo B. Flushing and slurred speech C. Cephalalgia and blurred vision D. Syncope and slow ventricular rate

D In complete atrioventricular block, the ventricles take over the pacemaker function in the heart but at a much slower rate than that of the SA node. As a result there is decreased cerebral circulation, causing syncope.

Which of the following classes of medications maximizes cardiac performance in clients with heat failure by increasing ventricular contractility? A. Beta-adrenergic blockers B. Calcium channel blockers C. Diuretics D. Inotropic agents

D Inotropic agents are administered to increase the force of the heart's contractions, thereby increasing ventricular contractility and ultimately increasing cardiac output. Beta-adrenergic blockers and calcium channel blockers decrease the heart rate and ultimately decrease the workload of the heart. Diuretics are administered to decrease the overall vascular volume, also decreasing the workload of the heart.

Which of the following symptoms usually signifies rapid expansion and impending rupture of an abdominal aortic aneurysm? A. Abdominal pain B. Absent pedal pulses C. Angina D. Lower back pain

D Lower back pain results from expansion of the aneurysm. The expansion applies pressure in the abdominal cavity, and the pain is referred to the lower back. Abdominal pain is most common symptom resulting from impaired circulation. Absent pedal pulses are a sign of no circulation and would occur after a ruptured aneurysm or in peripheral vascular disease. Angina is associated with atherosclerosis of the coronary arteries.

What is the primary reason for administering morphine to a client with myocardial infarction? A. To sedate the client B. To decrease the client's pain C. To decrease the client's anxiety D. To decrease oxygen demand on the client's heart

D Morphine is administered because it decreases myocardial oxygen demand. Morphine will also decrease pain and anxiety while causing sedation, but isn't primarily given for those reasons.

Following an acute myocardial infarction, a previously healthy 67-year-old patient develops clinical manifestations of heart failure. The nurse anticipates discharge teaching will include information about a. digitalis preparations, such as digoxin (Lanoxin). b. calcium-channel blockers, such as diltiazem (Cardizem). c. -adrenergic agonists, such as dobutamine (Dobutrex). d. angiotensin-converting enzyme (ACE) inhibitors, such as captopril (Capoten).

D Rationale: ACE-inhibitor therapy is currently recommended to prevent the development of heart failure in patients who have had a myocardial infarction and as a first-line therapy for patients with chronic heart failure. Digoxin therapy for heart failure is no longer considered a first-line measure, and digoxin is added to the treatment protocol when therapy with other medications such as ACE-inhibitors, diuretics, and -adrenergic blockers is insufficient. Calcium-channel blockers are not generally used in the treatment of heart failure. The -adrenergic agonists such as dobutamine are administered through the IV route and are not used as initial therapy for heart failure.

The nurse is caring for a patient receiving IV furosemide (Lasix) 40 mg and enalapril (Vasotec) 5 mg PO bid for ADHF with severe orthopnea. When evaluating the patient response to the medications, the best indicator that the treatment has been effective is a. weight loss of 2 pounds overnight. b. improvement in hourly urinary output. c. reduction in systolic BP. d. decreased dyspnea with the head of the bed at 30 degrees.

D Rationale: Because the patient's major clinical manifestation of ADHF is orthopnea (caused by the presence of fluid in the alveoli), the best indicator that the medications are effective is a decrease in crackles. The other assessment data also may indicate that diuresis or improvement in cardiac output have occurred but are not as useful in evaluating this patient's response.

A hospitalized patient with heart failure has a new order for captopril (Capoten) 12.5 mg PO. After administering the first dose and teaching the patient about captopril, which statement by the patient indicates that teaching has been effective? a. "I will need to include more high-potassium foods in my diet." b. "I will expect to feel more short of breath for the next few days." c. "I will be sure to take the medication after eating something." d. "I will call for help when I need to get up to the bathroom."

D Rationale: Captopril can cause hypotension, especially after the initial dose, so it is important that the patient not get up out of bed without assistance until the nurse has had a chance to evaluate the effect of the first dose. The ACE inhibitors are potassium sparring, and the nurse should not teach the patient to increase sources of dietary potassium. Increased shortness of breath is expected with initiation of -blocker therapy for heart failure, not for ACE-inhibitor therapy. ACE inhibitors are best absorbed when taken an hour before eating.

The nurse plans discharge teaching for a patient with chronic heart failure who has prescriptions for digoxin (Lanoxin), hydrochlorothiazide (HydroDIURIL), and a potassium supplement. Appropriate instructions for the patient include a. avoid dietary sources of potassium because too much can cause digitalis toxicity. b. take the pulse rate daily and never take digoxin if the pulse is below 60 beats/min. c. take the hydrochlorothiazide before bedtime to maximize activity level during the day. d. notify the health care provider immediately if nausea or difficulty breathing occurs.

D Rationale: Difficulty breathing is an indication of acute decompensated heart failure and suggests that the medications are not achieving the desired effect. Nausea is an indication of digoxin toxicity and should be reported so that the provider can assess the patient for toxicity and adjust the digoxin dose, if necessary. Digoxin toxicity is potentiated by hypokalemia, rather than hyperkalemia. Patients should be taught to check their pulse daily before taking the digoxin and, if the pulse is less than 60, to call their provider before taking the digoxin. Diuretics should be taken early in the day to avoid sleep disruption.

A 55-year-old patient with inoperable coronary artery disease and end-stage heart failure asks the nurse whether heart transplant is a possible therapy. The nurse's response to the patient will be based on the knowledge that a. heart transplants are experimental surgeries that are not covered by most insurance. b. the patient is too old to be placed on the transplant list. c. the diagnoses and symptoms indicate that the patient is not an appropriate candidate. d. candidacy for heart transplant depends on many factors.

D Rationale: Indications for a heart transplant include inoperable coronary artery disease and refractory end-stage heart failure, but other factors such as coping skills, family support, and patient motivation to follow the rigorous post-transplant regimen are also considered. Heart transplants are not considered experimental; rather, transplantation has become the treatment of choice for patients who meet the criteria. The patient is not too old for a transplant. The patient's diagnoses and symptoms indicate that the patient may be an appropriate candidate for a heart transplant.

When teaching the patient with heart failure about a 2000-mg sodium diet, the nurse explains that foods to be restricted include a. eggs and other high-cholesterol foods. b. canned and frozen fruits. c. fresh or frozen vegetables. d. milk, yogurt, and other milk products

D Rationale: Milk and yogurt naturally contain a significant amount of sodium, and intake of these should be limited for patients on a diet that limits sodium to 2000 mg daily. Other milk products, such as processed cheeses, have very high levels of sodium and are not appropriate for a 2000-mg sodium diet. The other foods listed have minimal levels of sodium and can be eaten without restriction.

Which of the following diagnostic tools is most commonly used to determine the location of myocardial damage? A. Cardiac catheterization B. Cardiac enzymes C. Echocardiogram D. Electrocardiogram

D The ECG is the quickest, most accurate, and most widely used tool to determine the location of myocardial infarction. Cardiac enzymes are used to diagnose MI but can't determine the location. An echocardiogram is used most widely to view myocardial wall function after an MI has been diagnosed. Cardiac catheterization is an invasive study for determining coronary artery disease and may also indicate the location of myocardial damage, but the study may not be performed immediately.

In which of the following disorders would the nurse expect to assess sacral edema in bedridden client? A. DM B. Pulmonary emboli C. Renal failure D. Right-sided heart failure

D The most accurate area on the body to assed dependent edema in a bedridden client is the sacral area. Sacral, or dependent, edema is secondary to right-sided heart failure. Diabetes mellitus, pulmonary emboli, and renal disease aren't directly linked to sacral edema.

Dyspnea, cough, expectoration, weakness, and edema are classic signs and symptoms of which of the following conditions? A. Pericarditis B. Hypertension C. Obliterative D. Restrictive

D These are the classic symptoms of heart failure. Pericarditis is exhibited by a feeling of fullness in the chest and auscultation of a pericardial friction rub. Hypertension is usually exhibited by headaches, visual disturbances and a flushed face. Myocardial infarction causes heart failure but isn't related to these symptoms.

Which of the following conditions is most closely associated with weight gain, nausea, and a decrease in urine output? A. Angina pectoris B. Cardiomyopathy C. Left-sided heart failure D. Right-sided heart failure

D Weight gain, nausea, and a decrease in urine output are secondary effects of right-sided heart failure. Cardiomyopathy is usually identified as a symptom of left-sided heart failure. Left-sided heart failure causes primarily pulmonary symptoms rather than systemic ones. Angina pectoris doesn't cause weight gain, nausea, or a decrease in urine output.

The nurse expects that a client with mitral stenosis would demonstrate symptoms associated with congestion in the: A. Aorta B. Right atrium C. Superior vena cava D. Pulmonary circulation

D When mitral stenosis is present, the left atrium has difficulty emptying its contents into the left ventricle. Hence, because there is no valve to prevent backward flow into the pulmonary vein, the pulmonary circulation is under pressure.

Which of the following treatments is the definitive one for a ruptured aneurysm? A. Antihypertensive medication administration B. Aortogram C. Beta-adrenergic blocker administration D. Surgical intervention

D When the vessel ruptures, surgery is the only intervention that can repair it. Administration of antihypertensive medications and beta-adrenergic blockers can help control hypertension, reducing the risk of rupture. An aortogram is a diagnostic tool used to detect an aneurysm.

A patient describes nonradiating substernal chest pain precipitated by climbing three flights of stairs. If the patient has taken one sublingual NTG tablet and asks what to do next if the pain is unrelieved, the nurse's best response would be A. "Call 911." B. "Drive to the emergency department at once." C. "I'm not qualified to advise you about this. Let me call your doctor." D. "Lie down and take another NTG tablet 5 minutes after the first tablet.

D. "Lie down and take another NTG tablet 5 minutes after the first tablet."

Which of the following information should be given to a patient before taking hydrochlorothiazide for control of hypertension? A. "You can develop a slower pulse rate." B. "You might notice some swelling in your feet." C. "You can develop shortness of breath or a cough." D. "Your diet should include foods high in potassium."

D. "Your diet should include foods high in potassium."

Which of the following changes in vital signs would most strongly suggest cardiac tamponade? A. Tachypnea B. Bradycardia C. Muffled heart sounds D. A narrowing pulse pressure

D. A narrowing pulse pressure

If a patient is being given a drug that blocks the action of the sympathetic division of the autonomic nervous system, the patient should be monitored for which of the following cardiovascular clinical manifestations? A. Increased heart rate and increased blood pressure B. Increased heart rate and decreased blood pressure C. Decreased heart rate and increased blood pressure D. Decreased heart rate and decreased blood pressure

D. Decreased heart rate and decreased blood pressure

If a patient self-administers 325 mg of acetylsalicylic acid (aspirin) daily, the health care provider should assess for which of the following most common side effects of salicylate therapy? A. Skin tears B. Hypothermia C. Hepatotoxicity D. GI distress

D. GI distress

If a patient who has experienced an MI develops left ventricular heart failure, for what sign of poor organ perfusion should a nurse remain alert? A. Headache B. Hypertension C. Heart rate of 55 to 60 beats/min D. Urine output of less than 30 ml/hour

D. Urine output of less than 30 ml/hour

An 80-year-old patient who lives in a skilled nursing facility and likes to walk is taking a medication that lowers blood pressure by dilating blood vessels. The most appropriate nursing action for this patient is to A. monitor blood pressure every day. B. order complete bed rest. C. monitor intake and output. D. assist the patient when ambulating.

D. assist the patient when ambulating.

The primary purpose of a soft, high-fiber diet immediately following a myocardial infarction (MI) is to A. promote easy digestion. B. lower cholesterol levels. C. maintain bowel health to decrease gas. D. create a high-bulk, soft stool to minimize Valsalvar maneuver.

D. create a high-bulk, soft stool to minimize Valsalva maneuver.

Etiology of heart failure after MI is related to the A. increased myocardial workload. B. increased oxygen demands of the myocardium. C. inability of the heart chambers to fill adequately. D. impairment of the contractile function of the ventricle.

D. impairment of the contractile function of the ventricle.

f a patient with heart failure is being treated with digoxin and has developed hypokalemia, the nurse should A. administer digoxin twice daily. B. reduce the digoxin dose to every other day. C. administer an IV bolus of potassium. D. monitor the patient for toxic effects that can occur at normal doses.

D. monitor the patient for toxic effects that can occur at normal doses.

The nurse monitors the patient receiving treatment for acute decompensated heart failure with the knowledge that marked hypotension is most likely to occur with the IV administration of A. furosemide (Lasix). B. milrinone (Primacor). C. nitroglycerin (Tridil). D. nitroprusside (Nipride).

D. nitroprusside (Nipride).

If a patient with dilated cardiomyopathy has an atrial fibrillation that has been unresponsive to drug therapy for several days, the nurse should anticipate that further treatment of the patient will require A. carotid massage. B. immediate electrical cardioversion. C. a transvenous implantable cardioverter-defibrillator. D. treatment with anticoagulants before additional treatment.

D. treatment with anticoagulants before additional treatment.

A patient is being discharged after the insertion of a permanent pacemaker. Which statement made by the patient indicates an understanding regarding appropriate self-care? A. "Every morning I will perform arm and shoulder stretches." B. "Each day I'll take my pulse and record it in a log." C. "I'll have to get rid of my microwave oven." D. "I won't be able to use my electric blanket anymore."

Each day I'll take my pulse and record it in a log

A patient who has experienced atrial fibrillation for the past 3 days is admitted to the cardiac care unit. In addition to administering an antidysrhythmia medication, the healthcare provider should anticipate which of these orders? A. Initiate a heparin infusion B. Give atropine IV push C. Prepare for immediate cardioversion D. Prepare the patient for AV node ablation

Heparin Because blood tends to pool and clot in the fibrillating atria, patients with atrial fibrillation are at high risk for embolic stroke, so heparin will be given.

With peripheral arterial insufficiency, leg pain during rest can be reduced by: a. Elevating the limb above heart level b. Lowering the limb so it is dependent c. Massaging the limb after application of cold compresses d. Placing the limb in a plane horizontal to the body

Lowering the limb so it is dependent

Mike, a 43-year old construction worker, has a history of hypertension. He smokes two packs of cigarettes a day, is nervous about the possibility of being unemployed, and has difficulty coping with stress. His current concern is calf pain during minimal exercise that decreased with rest. The nurse assesses Mike's symptoms as being associated with peripheral arterial occlusive disease. The nursing diagnosis is probably: a. Alteration in tissue perfusion related to compromised circulation b. Dysfunctional use of extremities related to muscle spasms c. Impaired mobility related to stress associated with pain d. Impairment in muscle use associated with pain on exertion.

a

The nurse finds a 76-year-old client's point of maximum impulse farther away from the fifth intercostal space, midclavicular line. What does this finding suggest to the nurse? a. The client has hypertension. b. The client had pneumonia. c. The client has benign prostatic hypertrophy. d. The client had a heart attack.

a

A patient with chronic HF and atrial fibrillation is treated with a digitalis glycoside and a loop diuretic. To prevent possible complications of this combination of drugs, what does the nurse need to do (select all that apply)? a. Monitor serum potassium levels b. teach the patient how to take a pulse rate. c. keep an accurate measure of intake and output d. Teach the patient about dietary restriction or potassium e. Withhold digitalis and notify health care provider if heart rate is irregular

a & b Rationale: Hypokalemia, which can be caused by the use of potassium-depleting diuretics (e.g., thiazides, loop diuretics), is one of the most common causes of digitalis toxicity. Low serum levels of potassium enhance the action of digitalis, causing a therapeutic dose to achieve toxic levels. Hypokalemia can also precipitate dysrhythmias. Monitoring the serum potassium levels of patients receiving digitalis preparations and potassium-depleting diuretics is essential. Patients taking digitalis preparations should be taught how to measure their pulse rate because bradycardia and atrioventricular blocks are late signs of digitalis toxicity. In addition, patients should know what pulse rate would necessitate a call to the health care provider.

Your patient is diagnosed with peripheral arterial disease and asks you what this disease is and what causes it? Which response is most appropriate? a) PAD is the thickening of arterial walls that results in gradual narrowing. This thickening and narrowing is related to arterial wall damage and inflammation. b) PAD is the thinning of arterial walls that results in gradual weakening. This weakening is related to high blood pressure. c) Let me get your doctor to explain this to you. d) PAD is the hardening of the arterial walls that results in thickening and eventual closure of the vessel.

a) PAD is the thickening of arterial walls that results in gradual narrowing. This thickening and narrowing is related to arterial wall damage and inflammation

A patient with PAD has a nursing diagnosis of ineffective peripheral tissue perfusion. Appropriate teaching for the patient includes instructions to (select all that apply). a) keep legs and feet warm b) walk at least 30 min/day to the point of discomfort c) apply cold compresses when the legs become swollen d) use nicotine replacement therapy as a substitute for smoking e) inspect lower extremities for pulses, temperature, and any swelling

a, b, and e

A patient admitted with heart failure appears very anxious and complains of shortness of breath. Which nursing actions would be appropriate to alleviate this patient's anxiety (select all that apply)? a. Administer ordered morphine sulfate. b. Position patient in a semi-Fowler's position. c. Position patient on left side with head of bed flat. d. Instruct patient on the use of relaxation techniques. e. Use a calm, reassuring approach while talking to patient.

a, b, d, & e Morphine sulfate reduces anxiety and may assist in reducing dyspnea. The patient should be positioned in semi-Fowler's position to improve ventilation that will reduce anxiety. Relaxation techniques and a calm reassuring approach will also serve to reduce anxiety.

The patient has heart failure (HF) with an ejection fraction of less than 40%. What core measures should the nurse expect to include in the plan of care for this patient (select all that apply)? a. Left ventricular function is documented. b. Controlling dysrhythmias will eliminate HF. c. Prescription for digoxin (Lanoxin) at discharge d. Prescription for angiotensin-converting enzyme (ACE) inhibitor at discharge e. Education materials about activity, medications, weight monitoring, and what to do if symptoms worsen

a, d, & e The Joint Commission has identified these three core measures for heart failure patients. Although controlling dysrhythmias will improve CO and workload, it will not eliminate HF. Prescribing digoxin for all HF patients is no longer done because there are newer effective drugs and digoxin toxicity occurs easily related to electrolyte levels and the therapeutic range must be maintained.

A nurse is assessing the neurovascular status of a client who returned to the surgical nursing unit 4 hours ago after undergoing aortoiliac bypass graft. The affected leg is warm, and the nurse notes redness and edema. The pedal pulse is palpable and unchanged from admission. The nurse interprets that the neurovascular status is: a. Normal because of the increased blood flow through the leg b. Slightly deteriorating and should be monitored for another hour c. Moderately impaired, and the surgeon should be called. d. Adequate from the arterial approach, but venous complications are arising.

a. An expected outcome of surgery is warmth, redness, and edema in the surgical extremity because of increased blood flow. Options 2, 3, and 4 are incorrect interpretations.

A patient with a recent diagnosis of heart failure has been prescribed furosemide (Lasix) in an effort to physiologically do what for the patient? a. Reduce preload. b. Decrease afterload. c. Increase contractility. d. Promote vasodilation.

a. Reduce preload. Diuretics such as furosemide are used in the treatment of HF to mobilize edematous fluid, reduce pulmonary venous pressure, and reduce preload. They do not directly influence afterload, contractility, or vessel tone.

The patient with chronic heart failure is being discharged from the hospital. What information should the nurse emphasize in the patient's discharge teaching to prevent progression of the disease to ADHF? a. Take medications as prescribed. b. Use oxygen when feeling short of breath. c. Only ask the physician's office questions. d. Encourage most activity in the morning when rested.

a. Take medications as prescribed. The goal for the patient with chronic HF is to avoid exacerbations and hospitalization. Taking the medications as prescribed along with nondrug therapies such as alternating activity with rest will help the patient meet this goal. If the patient needs to use oxygen at home, it will probably be used all the time or with activity to prevent respiratory acidosis. Many HF patients are monitored by a care manager or in a transitional program to assess the patient for medication effectiveness and monitor for patient deterioration and encourage the patient. This nurse manager can be asked questions or can contact the health care provider if there is evidence of worsening HF.

The nurse notes a client's PR interval upon ECG is greater than 0.20 seconds. What can this information suggest to the nurse? a. The client's sinoatrial node is not pacing correctly. b. There is an elongated ventricular depolarization. c. It takes longer for the blood to leave the lungs and return to the heart. d. The client elongated ventricular is filling.

a. The client's sinoatrial node is not pacing correctly.

The nurse recognizes that primary manifestations of systolic failure include: a. decreased EF and increased PAWP b. decreased PAWP and increased EF. c. decreased pulmonary hypertension associated with normal EF d. decreased afterload and decreased left ventricular end-diastolic pressure

a. decreased EF and increased PAWP Rationale: Systolic heart failure results in systolic failure in the left ventricle (LV). The LV loses its ability to generate enough pressure to eject blood forward through the aorta. This results in increased pulmonary artery wedge pressure (PAWP). The hallmark of systolic failure is a decrease in the left ventricular ejection fraction (EF).

Examination of a patient in a supine position reveals distended jugular veins from the base of the neck to the angle of the jaw. This finding indicates: a. decreased venous return. b. increased central venous pressure. c. increased pulmonary artery capillary pressure. d. left-sided heart failure.

b

Which technique is considered the gold standard for diagnosing DVT? a. Ultrasound imaging b. Venography c. MRI d. Doppler flow study

b

With peripheral arterial insufficiency, leg pain during rest can be reduced by: A. Elevating the limb above heart level B. Lowering the limb so it is dependent C. Massaging the limb after application of cold compresses D. Placing the limb in a plane horizontal to the body

b

Which of the following actions is the first priority care for a client exhibiting signs and symptoms of coronary artery disease? A. Decrease anxiety B. Enhance myocardial oxygenation C. Administer sublignual nitroglycerin D. Educate the client about his symptoms

b Enhancing mocardial oxygenation is always the first priority when a client exhibits signs and symptoms of cardiac compromise. Without adequate oxygen, the myocardium suffers damage. Sublingual nitorglycerin is administered to treat acute angina, but its administration isn't the first priority. Although educating the client and decreasing anxiety are important in care delivery, nether are priorities when a client is compromised.

Patients with a heart transplantation are at risk for which complications in the first year after transplantation (select all that apply)? a. cancer b. infection c. rejection d. vasculopathy e. sudden cardiac death

b, c, & e Rationale: A variety of complications can occur after heart transplantation. In the first year after transplantation, the major causes of death are acute rejection and infection. Heart transplant recipients also are at risk for sudden cardiac death. Later, malignancy (especially lymphoma) and cardiac vasculopathy (accelerated CAD) are major causes of death.

The nurse is preparing to administer digoxin to a patient with heart failure. In preparation, laboratory results are reviewed with the following findings: sodium 139 mEq/L, potassium 5.6 mEq/L, chloride 103 mEq/L, and glucose 106 mg/dL. What should the nurse do next? a. Withhold the daily dose until the following day. b. Withhold the dose and report the potassium level. c. Give the digoxin with a salty snack, such as crackers. d. Give the digoxin with extra fluids to dilute the sodium level.

b. Withhold the dose and report the potassium level. The normal potassium level is 3.5 to 5.0 mEq/L. The patient is hyperkalemic, which makes the patient more prone to digoxin toxicity. For this reason, the nurse should withhold the dose and report the potassium level. The physician may order the digoxin to be given once the potassium level has been treated and decreases to within normal range.

A nurse is caring for a client who had a percutaneous insertion of an inferior vena cava filter and was on heparin therapy before surgery. The nurse would inspect the surgical site most closely for signs of: a. Thrombosis and infection b. Bleeding and infection c. Bleeding and wound dehiscence. d. Wound dehiscence and evisceration.

b. After inferior vena cava insertion, the nurse inspects the surgical site for bleeding and signs and symptoms of infection. Otherwise, care is the same as for any post-op client.

A patient with a diagnosis of heart failure has been started on a nitroglycerin patch by his primary care provider. What should this patient be taught to avoid? a. High-potassium foods b. Drugs to treat erectile dysfunction c. Nonsteroidal antiinflammatory drugs d. Over-the-counter H2 -receptor blockers

b. Drugs to treat erectile dysfunction The use of erectile drugs concurrent with nitrates creates a risk of severe hypotension and possibly death. High-potassium foods, NSAIDs, and H2-receptor blockers do not pose a risk in combination with nitrates.

What should the nurse recognize as an indication for the use of dopamine (Intropin) in the care of a patient with heart failure? a. Acute anxiety b. Hypotension and tachycardia c. Peripheral edema and weight gain d. Paroxysmal nocturnal dyspnea (PND)

b. Hypotension and tachycardia Dopamine is a β-adrenergic agonist whose inotropic action is used for treatment of severe heart failure accompanied by hemodynamic instability. Such a state may be indicated by tachycardia accompanied by hypotension. PND, anxiety, edema, and weight gain are common signs and symptoms of heart failure, but these do not necessarily warrant the use of dopamine.

A client comes to the outpatient clinic and tells the nurse that he has had legs pains that begin when he walks but cease when he stops walking. Which of the following conditions would the nurse assess for? a. An acute obstruction in the vessels of the legs b. Peripheral vascular problems in both legs c. Diabetes d. Calcium deficiency

b. Intermittent claudication is a condition that indicates vascular deficiencies in the peripheral vascular system. If an obstruction were present, the leg pain would persist when the client stops walking. Low calcium levels may cause leg cramps but would not necessarily be related to walking.

A nurse is assessing a client with an abdominal aortic aneurysm. Which of the following assessment findings by the nurse is probably unrelated to the aneurysm? a. Pulsatile abdominal mass b. Hyperactive bowel sounds in that area c. Systolic bruit over the area of the mass d. Subjective sensation of "heart beating" in the abdomen.

b. Not all clients with abdominal aortic aneurysms exhibit symptoms. Those who do describe a feeling of the "heart beating" in the abdomen when supine or be able to feel the mass throbbing. A pulsatile mass may be palpated in the middle and upper abdomen. A systolic bruit may be auscultated over the mass. Hyperactive bowel sounds are not related specifically to an abdominal aortic aneurysm.

In preparation for discharge of a client with arterial insufficiency and Raynaud's disease, client teaching instructions should include: a. Walking several times each day as an exercise program. b. Keeping the heat up so that the environment is warm c. Wearing TED hose during the day d. Using hydrotherapy for increasing oxygenation

b. The client's instructions should include keeping the environment warm to prevent vasoconstriction. Wearing gloves, warm clothes, and socks will also be useful when preventing vasoconstriction, but TED hose would not be therapeutic. Walking would most likely increase pain.

A 24-year old man seeks medical attention for complaints of claudication in the arch of the foot. A nurse also notes superficial thrombophlebitis of the lower leg. The nurse would next assess the client for: a. Familial tendency toward peripheral vascular disease b. Smoking history c. Recent exposures to allergens d. History of insect bites

b. The mixture of arterial and venous manifestations (claudication and phlebitis, respectively) in the young male client suggests Buerger's disease. This is an uncommon disorder characterized by inflammation and thrombosis of smaller arteries and veins. This disorder typically is found in young adult males who smoke. The cause is not known precisely but is suspected to have an autoimmune component.

A client who has been receiving heparin therapy also is started on warfarin sodium (coumadin). The client asks the nurse why both medications are being administered. In formulating a response, the nurse incorporates the understanding that warfarin sodium: a. Stimulates the breakdown of specific clotting factors by the liver, and it takes 2-3 days for this is exhibit an anticoagulant effect. b. Inhibits synthesis of specific clotting factors in the liver, and it takes 3 to 4 days for this medication to exert an anticoagulation effect. c. Stimulates production of the body's own thrombolytic substances, but it takes 2-4 days for it to begin. d. Has the same mechanism action of heparin, and the crossover time is needed for the serum level of warfarin sodium to be therapeutic.

b. Warfarin sodium works in the liver and inhibits synthesis of four vitamin K-dependent clotting factors (X, IX, VII, and II), but it takes 3 to 4 days before the therapeutic effect of warfarin is exhibited.

Which are probable clinical findings in a person with an acute lower extremity VTE (select all that apply)? a.Pallor and coolness of foot and calf b.Mild to moderate calf pain and tenderness c.Grossly diminished or absent pedal pulses d.Unilateral edema and induration of the thigh e.Palpable cord along a superficial varicose vein

b.Mild to moderate calf pain and tenderness d.Unilateral edema and induration of the thigh The patient with lower extremity venous thromboembolism (VTE) may or may not have unilateral leg edema, extremity pain, a sense of fullness in the thigh or calf, paresthesias, warm skin, erythema, or a systemic temperature greater than 100.4 F (38 C). If the calf is involved, it may be tender to palpation.

A patient is admitted to the hospital with a diagnosis of abdominal aortic aneurysm. Which signs and symptoms would suggest that his aneurysm has ruptured? a.Sudden shortness of breath and hemoptysis b.Sudden, severe low back pain and bruising along his flank c.Gradually increasing substernal chest pain and diaphoresis d.Sudden, patchy blue mottling on feet and toes and rest pain

b.Sudden, severe low back pain and bruising along his flank The clinical manifestations of a ruptured abdominal aortic aneurysm include severe back pain, back or flank ecchymosis (Grey Turner's sign), and hypovolemic shock (tachycardia, hypotension, pale clammy skin, decreased urine output, altered level of consciousness, and abdominal tenderness).

The patient at highest risk for venous thromboembolism (VTE) is a.a 62-year-old man with spider veins who is having arthroscopic knee surgery. b.a 32-year-old woman who smokes, takes oral contraceptives, and is planning a trip to Europe. c.a 26-year-old woman who is 3 days postpartum and received maintenance IV fluids for 12 hours during her labor. d.an active 72-year-old man at home recovering from transurethral resection of the prostate for benign prostatic hyperplasia.

b.a 32-year-old woman who smokes, takes oral contraceptives, and is planning a trip to Europe. Three important factors (called Virchow's triad) in the etiology of venous thrombosis are (1) venous stasis, (2) damage of the endothelium (inner lining of the vein), and (3) hypercoagulability of the blood. Patients at risk for venous thrombosis usually have predisposing conditions for these three disorders (see Table 38-8). The 32-year-old woman has the highest risk: long trips without adequate exercise (venous stasis), tobacco use, and use of oral contraceptives. Note: The likelihood of hypercoagulability of blood is increased in women older than 35 years who use tobacco.

A nurse, explaining the cardiac circulation to cardiac rehabilitation clients, wants to include the oxygenation of the heart muscle. Which of the following structures carries deoxygenated blood to the lungs? a. Right main coronary vein b. Pulmonary vein c. Pulmonary artery d. Great cardiac vein

c

The healthcare provider is caring for a patient with a diagnosis of mitral stenosis. When auscultating the patient's chest, which type of heart sound should the healthcare provider expect to hear? Please choose from one of the following options. a. Systolic ejection murmur b. Midsystolic murmur c. Diastolic murmur d. Pansystolic murmur

c

When caring for a patient who has started anticoagulant therapy with warfarin (Coumadin), the nurse knows not to expect therapeutic benefits for: a. At least 12 hours b. The first 24 hours c. 2-3 days d. 1 week

c

Medical treatment of coronary artery disease includes which of the following procedures? A. Cardiac catheterization B. Coronary artery bypass surgery C. Oral medication administration D. Percutaneous transluminal coronary angioplasty

c Oral medication administration is a noninvasive, medical treatment for coronary artery disease. Cardiac catheterization isn't a treatment but a diagnostic tool. Coronary artery bypass surgery and percutaneous transluminal coronary angioplasty are invasive, surgical treatments.

You are caring for a patient with ADHF who is receiving IV dobutamine (Dobutrex). You know that this drug is ordered because it (select all that apply): a. incerases SVR b. produces diuresis c. improves contractility d. dilates renal blood vessels e. works on the B1-receptors in the heart.

c & e Rationale: Dobutamine (Dobutrex) has a positive chronotropic effect and increases heart rate and improves contractility. It is a selective β-adrenergic agonist and works primarily on the β1-adrenergic receptors in the heart. It is frequently used in the short-term management of acute decompensated heart failure (ADHF).

ollowing teaching about medications for PAD, the nurse determines that additional instruction is necessary when the patient says, a) I should take one ASA a day to prevent clotting in my legs b) The lisinipril (Zestril) I use for my BP may help me walk further without pain c) I will need to have frequent blood tests to evaluate the effect of the oral anticoagulant I will be taking. d) Pletal should help me be able to increase my walking distance and keep clots from forming in my legs

c) I will need to have frequent blood tests to evaluate the effect of the oral anticoagulant I will be taking.

A client has been diagnosed with a myocardial infarction that has damaged a part of the right atrium. Which of the following could happen as a result of this damage? a. Increase in collateral circulation b. Nothing c. A change in the rate and rhythm of the client's heartbeat d. Onset of aortic regurgitation

c. A change in the rate and rhythm of the client's heartbeat

What is the priority assessment by the nurse caring for a patient receiving IV nesiritide (Natrecor) to treat heart failure? a. Urine output b. Lung sounds c. Blood pressure d. Respiratory rate

c. Blood pressure Although all identified assessments are appropriate for a patient receiving IV nesiritide, the priority assessment would be monitoring for hypotension, the main adverse effect of nesiritide.

Which of the following characteristics is typical of the pain associated with DVT? a. Dull ache b. No pain c. Sudden onset d. Tingling

c. DVT is associated with deep leg pain of sudden onset, which occurs secondary to the occlusion. A dull ache is more commonly associated with varicose veins. A tingling sensation is associated with an alteration in arterial blood flow. If the thrombus is large enough, it will cause pain.

A compensatory mechanism involved in HF that leads to inappropriate fluid retention and additional workload of the heart is: a. ventricular dilation b. ventricular hypertrophy c. neurohormonal response d. sympathetic nervous system activation

c. neurohormonal response Rationale: The following mechanisms in heart failure lead to inappropriate fluid retention and additional workload of the heart: activation of the renin-angiotensin-aldosterone system (RAAS) cascade and release of antidiuretic hormone from the posterior pituitary gland in response to low cerebral perfusion pressure that results from low cardiac output.

A 50-year-old woman weighs 95 kg and has a history of tobacco use, high blood pressure, high sodium intake, and sedentary lifestyle. When developing an individualized care plan for her, the nurse determines that the most important risk factors for peripheral artery disease (PAD) that need to be modified are a.weight and diet. b.activity level and diet. c.tobacco use and high blood pressure. d.sedentary lifestyle and high blood pressure.

c. tobacco use and high blood pressure Significant risk factors for peripheral artery disease include tobacco use, hyperlipidemia, elevated levels of high-sensitivity C-reactive protein, diabetes mellitus, and uncontrolled hypertension; the most important is tobacco use. Other risk factors include family history, hypertriglyceridemia, hyperuricemia, increasing age, obesity, sedentary lifestyle, and stress.

During the cardiac focused interview, the client tells the nurse he has smoked for 30 years. Which of the following would be appropriate for the nurse to respond to this client? a. High dose vitamin therapy reduces the risks of smoking. b. Exercise reduces the harmful effects of smoking. c. Do you also take recreational drugs? d. Smoking has been linked to high blood pressure and other heart problems.

d

The nurse is calculating the cardiac output for a client with a heart rate of 88 beats per minute. What other measurements does the nurse need to complete this calculation? a. Cardiac index b. QT interval c. T wave d. Stroke volume

d

A client is scheduled for a cardiac catheterization using a radiopaque dye. Which of the following assessments is most critical before the procedure? A. Intake and output B. Baseline peripheral pulse rates C. Height and weight D. Allergy to iodine or shellfish

d This procedure requires an informed consent because it involves injection of a radiopaque dye into the blood vessel. The risk of allergic reaction and possible anaphylaxis is serious and must be assessed before the procedure.

A significant cause of venous thrombosis is: a. Altered blood coagulation b. Stasis of blood c. Vessel wall injury d. All of the above

d all of the above

Following discharge teaching with a male patient with an AAA repair, the nurse determines that further instruction is needed when the patient says, a) I should avoid heavy lifting b) I may have some permanent sexual dysfunction as a result of the surgery c) I should maintain a low-fat and low-cholesterol diet to help keep the new graft open d) I should take the pulses in my extremities and let the doctor know if they get too fast or too slow.

d) I should take the pulses in my extremities and let the doctor know if they get too fast or too slow.

What is the leading cause of PAD? a) smoking b) hyperlipidemia c) hypertension d) atherosclerosis

d) atherosclerosis

The nurse is administering a dose of digoxin (Lanoxin) to a patient with heart failure (HF). The nurse would become concerned with the possibility of digitalis toxicity if the patient reported which symptom(s)? a. Muscle aches b. Constipation c. Pounding headache d. Anorexia and nausea

d. Anorexia and nausea Anorexia, nausea, vomiting, blurred or yellow vision, and cardiac dysrhythmias are all signs of digitalis toxicity. The nurse would become concerned and notify the health care provider if the patient exhibited any of these symptoms.

A stable patient with acute decompensated heart failure (ADHF) suddenly becomes dyspneic. Before positioning the patient on the bedside, what should the nurse assess first? a. Urine output b. Heart rhythm c. Breath sounds d. Blood pressure

d. Blood pressure The nurse should evaluate the blood pressure before dangling the patient on the bedside because the blood pressure can decrease as blood pools in the periphery and preload decreases. If the patient's blood pressure is low or marginal, the nurse should put the patient in the semi-Fowler's position and use other measures to improve gas exchange.

Beyond the first year after a heart transplant, the nurse knows that what is a major cause of death? a. Infection b. Acute rejection c. Immunosuppression d. Cardiac vasculopathy

d. Cardiac vasculopathy Beyond the first year after a heart transplant, malignancy (especially lymphoma) and cardiac vasculopathy (accelerated CAD) are the major causes of death. During the first year after transplant, infection and acute rejection are the major causes of death. Immunosuppressive therapy will be used for posttransplant management to prevent rejection and increases the patient's risk of an infection.

A male patient with a long-standing history of heart failure has recently qualified for hospice care. What measure should the nurse now prioritize when providing care for this patient? a. Taper the patient off his current medications. b. Continue education for the patient and his family. c. Pursue experimental therapies or surgical options. d. Choose interventions to promote comfort and prevent suffering

d. Choose interventions to promote comfort and prevent suffering. The central focus of hospice care is the promotion of comfort and the prevention of suffering. Patient education should continue, but providing comfort is paramount. Medications should be continued unless they are not tolerated. Experimental therapies and surgeries are not commonly used in the care of hospice patients.

After having an MI, the nurse notes the patient has jugular venous distention, gained weight, developed peripheral edema, and has a heart rate of 108/minute. What should the nurse suspect is happening? a. ADHF b. Chronic HF c. Left-sided HF d. Right-sided HF

d. Right-sided HF An MI is a primary cause of heart failure. The jugular venous distention, weight gain, peripheral edema, and increased heart rate are manifestations of right-sided heart failure.

Intravenous heparin therapy is ordered for a client. While implementing this order, a nurse ensures that which of the following medications is available on the nursing unit? a. Vitamin K b. Aminocaproic acid c. Potassium chloride d. Protamine sulfate

d. The antidote to heparin is protamine sulfate and should be readily available for use if excessive bleeding or hemorrhage should occu

A client is admitted with a venous stasis leg ulcer. A nurse assesses the ulcer, expecting to note that the ulcer: a. Has a pale colored base b. Is deep, with even edges c. Has little granulation tissue d. Has brown pigmentation around it.

d. Venous leg ulcers, also called stasis ulcers, tend to be more superficial than arterial ulcers, and the ulcer bed is pink. The edges of the ulcer are uneven, and granulation tissue is evident. The skin has a brown pigmentation from accumulation of metabolic waste products resulting from venous stasis. The client also exhibits peripheral edema. (options 1, 2, and 3 is due to tissue malnutrition; and thus us an arterial problem)


Set pelajaran terkait

Nursing Application: Antiparkinson Drugs

View Set

Clinical Chem Test Bank (Final Exam)

View Set

Henry VIII and the English Reformation

View Set